[obm-l] Vetores

2009-04-14 Por tôpico marcio aparecido
1-Determinar o Vetor V, ortogonal ao vetor U = (2,-3,-12), e colinear ao
vetor W=(-6,4,-2)

2-Os lados de um triângulo retângulos ABC reto em A medem 5, 12 e 13.
Cacular AB.AC+BA.BC+CA.CB.


Re: [obm-l] Re: [obm-l] Re: [obm-l] Congruência de Triângulos

2008-01-30 Por tôpico marcio aparecido
estou na oitava série, nesse periodo eu tentei umas 4, 5 vezes fazer!!


[obm-l] Teorema de Tales

2008-01-29 Por tôpico marcio aparecido
Olá pessoal.
Queria saber se alguem poderia me ajudar com uma demonstração do teorema de
Tales.
Tenho o livro fundamentos de matemática elementarmas não conseguir
entender a demonstração dele.achei ela muito complicada.
agradeço a ajuda.


Re: [obm-l] Re: [obm-l] Congruência de Triângulos

2008-01-29 Por tôpico marcio aparecido
não consegui demostrar


[obm-l] Congruência de Triângulos

2007-10-09 Por tôpico marcio aparecido
Como eu posso fazer para provar os casos ALA e LLL de congruência de
triângulos ??


Re: [obm-l] número irracional

2007-08-11 Por tôpico Marcio Cohen
Pessoal, muito cuidado com afirmacoes que nao vem acompanhada de prova ou
referencia. Assim como o comentario sobre a soma de dois transcendentes ser
tambem transcendente, a afirmacao feita na mensagem do Andre eh falsa.

Por exemplo, considere a = 2, b = log3/log2.

Por um lado, a eh claramente natural, e b eh irracional (de fato, se b=p/q,
p,q inteiros positivos, temos 2^p = 3^q que eh uma contradicao pois nenhuma
potencia de 3 eh par).

Por outro lado, a^b = 3 nao eh um numero irracional!

Abracos,
Marcio Cohen


On 8/11/07, André Smaira [EMAIL PROTECTED] wrote:

 vc já sabe q 3^(1/2)=sqrt(3) eh irracional e um numero natural elevado a
 um irracional é irracional

 Flickr agora em português. Você clica, todo mundo vê. Saiba 
 maishttp://br.rd.yahoo.com/taglines/flickr/*http://www.flickr.com.br/.




Re: [obm-l] Uma boa de geometria - CALMA !!!

2007-07-31 Por tôpico Marcio Cohen
Douglas,

Você certamente fez a parte difícil da questão e merece 100% dos créditos
por isso. Eu tinha feito uma solução por complexos para a questão da Eureka
na aula de treinamento da imo, mas a sua é muito mais legal!!

Para provar o detalhe final da sua solução, minha estratégia padrão é:

Seja a=exp(iA), b=exp(iB), c=exp(iC). Então, abc = -1 e como
exp(ix)+exp(-ix) = 2cosx:

(cosA)^2 + (cosB)^2 + (cosC)^2 = (1/4)*(a^2 + 1/a^2 + b^2 + 1/b^2 + c^2 +
1/c^2 + 6)
= (1/4)*(a^2 + b^2 + c^2 + (bc)^2 + (ab)^2 + (ac)^2+6);


8cosA*cosB*cosC = (a+1/a)(b+1/b)(c+1/c) = -(1+a^2)(1+b^2)(1+c^2) =
-(1+a^2+b^2+c^2+(ab)^2+(ac)^2+(bc)^2+1).

Substituindo uma na outra, 8cosA*cosB*cosC = -(2+4*( (cosA)^2 + (cosB)^2 +
(cosC)^2 - 6), ou seja,
(cosA)^2 + (cosB)^2 + (cosC)^2 = 1 - 2cosAcosBcosC

Abraços,
Marcio Cohen

On 7/30/07, Douglas Ribeiro Silva [EMAIL PROTECTED] wrote:

 Olá Nehab!

 Primeiramente gostaria de expressar minha satisfação do problema ter
 de fato chamado sua atenção e do Rogério Ponce. Já participo da
 lista(não muito ativamente) há um bom tempo e percebo que assim como
 eu, vocês gostam muito de geometria.

 O problema na verdade veio da minha cabeça, mas foi inspirado em um
 problema proposto na última(ou penúltima) Eureka. Originalmente o
 problema pedia para mostrar que XYZ estão alinhados se e somente se
 cosA*cosB*cosC = -3/8. Então pensei em me inspirar nos chineses, que
 gostavam de resolver teoremas usando áreas, e pensei em zerar a área
 do triangulo XYZ para chegar na tão esperada relação.

 Por um lado estou satisfeito, pois consegui chegar em uma expressão
 que relaciona as áreas corretamente, por outro estou frustrado pois
 não consigo fazer a última passagem, que certamente exige uma
 fatoração ou algo do tipo que não estou conseguindo enxergar.

 A relação que eu cheguei foi S(XYZ) = S(ABC)*[7 - 4((cosA)^2 +
 (cosB)^2 + (cosC)^2)].
 Note que a relação é válida nos casos mais triviais em que o triangulo
 é equilatero, retangulo(que, como o Rogerio falou, são respectivamente
 1:4 e 1:3) ou isosceles com angulo de 120 graus(basta fazer um desenho
 para ver que a área dá zero). Notem que a relação pedida no problema
 da Eureka é satisfeita para este triangulo isosceles.

 Aos curiosos que querem saber como eu cheguei nessa relação, segue a
 idéia abaixo:

 Construam o triangulo ABC e suas respectivas reflexões XYZ.
 Observe que S(XYZ) = [S(ABC) + S(BCX) + S(ACY) + S(ABZ)] - S(AYZ) -
 S(XBZ) - S(XYC)

 S(ABC) = S(BCX) = S(ACY) = S(ABZ) por construção

 As áreas de AYZ XBZ e XYC podem ser somadas ou subtraídas, dependendo
 se os ângulos YAB = 3A, XBZ = 3B ou XCY = 3C forem maiores ou menores
 que 180 graus. Para esses triangulos vou usar que S(AYZ) =
 bc*sen(3A)/2, S(XBZ) = ac*sen(3B)/3 e S(XYC) = ab*sen(3C)/2.

 Então a relação passa a ser S(XYZ) = 4S(ABC) - bc*sen(3A)/2 -
 ac*sen(3B)/3 - ab*sen(3C)/2

 Agora substituímos sen(3X) = -4*[sen(X)]^3 + 3*sen(X) em todos e
 substituímos também bc/2 ac/2 e ab/2 respectivamente por
 S(ABC)/sen(A), S(ABC)/sen(B) e S(ABC)/sen(C), devido à mesma fórmula
 de área em função dos lados e do angulo para o triangulo original.

 Fazendo as devidas substituições acima, simplificamos os senos e
 ficamos com a relação da soma dos quadrados dos senos. Basta trocar
 [sen(X)]^2 por 1 - [cos(X)]^2 e chegamos em S(XYZ) = S(ABC)*[7 -
 4((cosA)^2 + (cosB)^2 + (cosC)^2)].

 Quando eu enviei o problema ainda não tinha chegado nesse resultado e
 achava que chegaria em uma expressão mais fácil de passar para o
 produto de cossenos.

 Qualquer ajuda para terminar o problema eu agradeço bastante e
 certamente darei os devidos créditos quando enviar a solução para a
 Eureka.

 Abraços, Douglas




 Em 30/07/07, Carlos Eddy Esaguy Nehab[EMAIL PROTECTED] escreveu:
 
   Oi, querido Ponce
 
   Naturalmente não se supunha (pelo menos eu) que a relação entre as
 áreas
  independesse do triângulo, mas mesmo assim, confesso que tentei vários
  caminhos e não encontrei uma solução simples para o problema.
 
   Eu esperava algo do tipo:  a razão entre as áreas é o quadrado do
 produto
  dos senos dos angulos, ou  coisa similar.  Embora tendo encontrado
 várias
  coisas curiosas sobre o maldito e interessante triângulo, tentando
 resolver
  o problema, não encontrei nada simples que merecesse ser publicado.
 
   E também confesso que imaginei que alguém mais inspirado conseguisse
 alguma
  expressão simples para a resposta.Resta aguardar  que quem propôs o
  problema informe se sabe alguma coisa (aliás hábito pouco praticado em
 nossa
  lista é informar a origem dos problemas propostos - e às vezes, a origem
 é
  bastante interessante).
 
   Eu realmente gosto desta informação pois tenho o hábito (e gosto) de
  mencionar a origem (e a solução) de qualquer problema que eu proponho,
 no
  mínimo para respeitar a história... e o trabalho alheio.
 
   Abraços,
   Nehab
 
   At 01:09 29/7/2007, you wrote:
 
  Ola' Douglas e colegas da lista,
   nao existe uma relacao fixa entre as 2 areas

Re: [obm-l] RES: [obm-l] Convergência/divergência de uma serie

2007-04-19 Por tôpico Marcio Cohen

Oi Arthur,

Na verdade, (1+1/n^(4/3))^(n^(4/3)) - e nao eh o mesmo que
(1+1/n^(4/3))^n - e^(3/4) pq o expoente 4/3 esta soh no n e nao no
(1+1/n^(4/3))^n..

Acho inclusive que essa série diverge, pois como 2^x  1+x*ln2 para x0, temos
  Soma ( 2^(1/n) - 1)  ln2*Soma (1/n) ...

Abraços,
Marcio



On 4/19/07, Artur Costa Steiner [EMAIL PROTECTED] wrote:



Eu encontrei uma solucao um tanto artesanal.

Partimos de lim (1 + 1/n)^n = e. Assim, temos tambem que  lim (1 +
1/n^(4/3))^(n^(4/3)) = e, o que eh o mesmo que dizer que
.
lim (1 + 1/n^(4/3))^n = e^(3/4).

Temos que e^(3/4)  (2,5)^(3/4) = (1 + 1,5)^(3/4)  1 + 1,5 * 3/4 = 2,125 
2 . Assim, para n suficientemente grande temos que

(1 + 1/n^(4/3))^n  2

Tomando a raiz enésima, vem

1 + 1/n^(4/3)  2^(1/n) e, portanto, 1/n^(4/3)  2^(1/n) - 1.

Para n suficientemente grande, temos portanto que

0  2^(1/n) - 1  1/n^(4/3)

Como 4/3 1, a serie Soma 1/n^(4/3) converge. Por comparacao, concluimos
entao que Soma ( 2^(1/n) - 1) converge,

Abracos

Artur


.

-Mensagem original-
De: [EMAIL PROTECTED] [mailto:[EMAIL PROTECTED] nome de
ralonso
Enviada em: quinta-feira, 19 de abril de 2007 12:50
Para: obm-l@mat.puc-rio.br
Assunto: Re: [obm-l] Convergência/divergência de uma serie


   A série começa com 2 e os termos vão diminuindo até zero,
assim dá para suspeitar que converge porque o termo geral tende a zero.
  Mas o termo geral tender a zero, não é uma condição suficiente para
convergência.  Precisamos de um critério, como o da comparação.
   Eu tentaria, de imediato, algo do tipo:
Pegaria uma série que eu sei que converge tal como
a_n =   1/(2^n-1), cuja conclusão se tira pela comparação com a série
geométrica,   e b_n = 2^(1/n) - 1 e calcularia
o limite a_n/b_n quando n - infinito.  Foi isso que você fez?

Ronaldo.

Artur Costa Steiner wrote:
 Achei a analise da convergencia/divergencia desta serie interessante:Soma
(n =1, oo) (2^(1/n) - 1)Conclui que converge.AbracosArtur






=
Instruções para entrar na lista, sair da lista e usar a lista em
http://www.mat.puc-rio.br/~nicolau/olimp/obm-l.html
=


Re: [obm-l] séries numéricas

2007-04-07 Por tôpico Marcio Cohen

Oi Cláudio,

Bem vindo a lista.

Uma sugestão é verificar que para qualquer função positiva decrescente f, (e
em particular para as duas funções que vc considerou),

Somatório_n=2..oo_f(n) converge se e somente se Integral_x=2..oo_f(x)
converge
(veja isso pela definição de integral ou pela comparação das áreas dos
gra'ficos).

No 1o caso, f(x) = 1/x/logx e a integral indefinida vale log(logx), que pode
ficar tão grande quanto se queira.

Para a funcao f(x)=1/x/log^r(x) com r1, a integral indefinida vale
-1/((logx)^(r-1))/(r-1), que tende ao valor finito +1/(r-1)/log2 quando x
tende a infinito.

Fica como exercício analisar a convergência da série cujo termo geral é
1/(logn)^(logn).

Abraços,
Marcio Cohen

On 4/7/07, Claudio Gustavo [EMAIL PROTECTED] wrote:


  Oi. Sou Claudio Gustavo e esta é a primeira vez que escrevo para esta
lista. Gostaria de alguma dica para demonstrar que a soma de n=2 até
infinito de 1/(n*logn) diverge e a soma 1/(n*(logn)^r), com r mairo que
1, converge. Tem alguma possibilidade de comparar com as somas harmônicas?
Pois a soma 1/n diverge e 1/(n^r) converge para r maior que 1.
  Obrigado.

__
Fale com seus amigos de graça com o novo Yahoo! Messenger
http://br.messenger.yahoo.com/



Re: [obm-l] integral

2007-04-07 Por tôpico Marcio Cohen

Aqui vai uma outra solução bem interessante para a integral I = int(0--+00)
(arctan(pi.x) - arctan(x))/x dx.

Ela se baseia na observação de que arctan(pi.x) - arctan(x) eh a integral de
1/(t^2+1) de x até pi.x (*).

Logo, a integral pedida pode ser calculada como um integral dupla:

I = Integral Dupla_x=0..oo_t=x..pix_(dtdx/(t^2+1)/x)

Trocando a ordem de integracao,

I = Integral Dupla_t=0..oo_x=t/pi..t_(dxdt/(t^2+1)/x)

E agora é fácil, pois Integral_x=t/pi..t_(dx/x) = lnt-ln(t/pi) = ln(pi) eh
constante, implicando

I = ln(pi)*Integral_t=0..oo_(dt/(t^2+1)) = ln(pi)*pi/2 pela observacao *.

Abracos,
Marcio Cohen

On 4/5/07, Demetrio Freitas [EMAIL PROTECTED] wrote:



Buenas,

Vamos começar pela fórmula da integral por partes:

int(a..b)(u dv) = uv(b)-uv(a) -int(a..b)(v du)

No caso, temos:
u = arctan(pi.x) - arctan(x)
v = ln(x)

int(0..+oo)( (arctan(pi.x) - arctan(x) )/x dx =

lim(x-oo)( (arctan(pi.x) - arctan(x) )*ln(x) -

lim(x-0)( (arctan(pi.x) - arctan(x)  )*ln(x) -

int(0..oo)( (Pi/(1+Pi^2*x^2)-1/(1+x^2) )*ln(x)) dx

O segundo limite é zero (basta olhar a expansão de
taylor para arctan(x)).

O primeiro limite também é zero. Uma forma de ver pode
ser:

(arctan(pi.x) - arctan(x))*ln(x) =  (arctan(pi.x) -
arctan(x))  /  (1/ln(x))

lim(x-oo) ((arctan(pi.x) - arctan(x)) / (1/ln(x)) =
LHospital =
lim(x-oo) (( Pi/(1+Pi^2*x^2) - 1/(1+x^2) ) *
ln(x)^2*x )  = 0

Então ficamos com:

int(0..+oo)( ( arctan(pi.x) - arctan(x) )/x dx =
int(0..oo)( -f(x) ) dx, Onde:
f(x) = ( Pi/(1+Pi^2*x^2) - 1/(1+x^2) ) * ln(x)

Agora vamos considerar a integral tomada entre -oo e
+oo, e lembrar que, para x E R0, ln(-x)=ln(x)+i*Pi.

Assim:
int(-oo..+oo)(f(x)) dx = 2*int(0..oo) f(x)  dx +
i*Pi*int(0..oo) Pi/(1+Pi^2*x^2) -1/(1+x^2)) dx

Bem,  int(-oo..+oo) f(x)  dx pode ser calculada por
resíduos. Depois, vc toma a parte real para calcular
int(0..oo) f(x)  dx =

f(x) tem dois pólos no semiplano complexo z=x+i*y com
y0, que são: z=i e z=i/Pi.

Res(z=i) = lim( x-i ) ( f(x)*(x-i) ) = -Pi/4
Res(z=i/Pi) = lim( x-i/Pi )( f(x)*(x-i/Pi) ) = Pi/4
+i/2*ln(Pi)

int(-oo..+oo) f(x) dx =2*Pi*i*( -Pi/4+Pi/4+i/2*
ln(Pi))

int(-oo..+oo) f(x) dx = -Pi*ln(Pi)

A integral pedida é então:

int(0..+oo) (arctan(pi.x) - arctan(x))/x dx =

int(0..oo) -f(x) dx = 1/2 * Pi * ln(Pi)

[]´s Demetrio



--- BRENER [EMAIL PROTECTED] escreveu:

 Ola, gostaria de uma ajudinha na integral

 int(0--+00) (arctan(pi.x) - arctan(x))/x dx


Re: [obm-l] duvida - Vietnam Undergraduate Mathemtics Competition 2001

2007-02-22 Por tôpico Marcio Cohen

Oi Marcelo,

A ideia eh que se x é autovalor de A, entao x^k eh autovalor de A^k, pois Au
= xu = (A^k)u=(x^k)u.
Como A^k = 0 e autovetores sao nao nulos, isso significa que x^k=0, ou seja,
x=0.

On 2/22/07, Marcelo Salhab Brogliato [EMAIL PROTECTED] wrote:


Olá Marcio,

se A é nilpotente, entao existe k, tal que: A^k = 0
A^k - sI = -sI  det(A^k - sI) = (-s)^n, onde n é a dimensao de A
assim, o unico autovalor de A^k é 0, pois é o unico que zera (-s)^n...

nao consegui provar que A tem os autovalores nulos =/
dps tento novamente
abracos
Salhab




- Original Message -
From: Marcio Cohen [EMAIL PROTECTED]
To: obm-l@mat.puc-rio.br
Sent: Monday, February 19, 2007 5:18 PM
Subject: Re: [obm-l] duvida - Vietnam Undergraduate Mathemtics Competition
2001


Se det(B) nao for 0, entao B admite inversa B^-1, e portanto podemos
escrever A=AB^-1+I. Logo, det(A-I) = det(AB^-1) = det(A)*... = 0 = 1
é autovalor de A (contradição!).

A gente chama uma matriz de nilpotente quando existe um inteiro k tal
que A^k = 0. Verifique que A é nilpotente sse seus autovalores são
todos nulos.

Abraços,
Marcio Cohen

On 2/19/07, Jhonata Ramos [EMAIL PROTECTED] wrote:
 Pessoal,
 tava olhando essa questão:

 Let be given two matrices A, B from M_n(R) such that A^2001 = 0 and AB
 = A+B. Show that det(B) = 0.

 Source VUMC 2001

 Vi uma solução que o cara fala o seguinte:

 A^2001=0 = A is nipoltent detA=0
 lemma: If X,Y commute, Y nilpotent
 then det(X+Y)=detX

 Gostaria de saber o que significa, nipoltent (não a tradução :)
 e se o lemma dele ali é verdadeiro,

 Forte abraço,
 Jhonata Emerick Ramos


=
 Instruções para entrar na lista, sair da lista e usar a lista em
 http://www.mat.puc-rio.br/~nicolau/olimp/obm-l.html

=


=
Instruções para entrar na lista, sair da lista e usar a lista em
http://www.mat.puc-rio.br/~nicolau/olimp/obm-l.html
=

=
Instruções para entrar na lista, sair da lista e usar a lista em
http://www.mat.puc-rio.br/~nicolau/olimp/obm-l.html
=



Re: [obm-l] simetria / putnam 87

2007-02-19 Por tôpico Marcio Cohen

Oi Jhonata,

Nesse caso, a motivação foi buscar limites de integração simétricos
(de -a até a), pois isso lhe permitirá ver mais facilmente idéias como
trocar u por -u ou integral de função ímpar de -a até a vale zero.

Como os limites vão de x = 2 até x = 4, i.e, de x = 3-1 até x=3+1, é
razoável você fazer a substituição de variáveis u = x - 3.

Outro exemplo resolvido:

I = Integral_de 0 a pi/2_ dx/(1+(tanx)^r).
Como a integral vai de x=0 a x=pi/2, eu vou fazer a troca u = x-pi/4,
de forma que u vai de -pi/4 até pi/4.
Como tanx = tan(u+pi/4) = (1+tanu)/(1-tanu), temos

I = Integral_de -pi/4 a pi/4_ (1-tanu)^r du/ ( (1-tanu)^r + (1+tanu)^r )
Trocando u por -u, como tan é uma função ímpar:
I = Integral_de -pi/4 a pi/4_ (1+tanu)^r du/ ( (1+tanu)^r + (1-tanu)^r ).

Somando as duas, obtemos 2I = Integral_de -pi/4 a pi/4_ du = pi/2, logo I=pi/4.

Você pode ler mais a respeito no livro Problem-Solving Through
Problems, de Loren Larson por exemplo...

Abraços,
Marcio Cohen



On 2/19/07, Jhonata Ramos [EMAIL PROTECTED] wrote:

Bom dia amigos da lista,

estava resolvendo(ou pelo menos tentando :) algumas questões e me
deparei com essa:

http://www.majorando.com/arquivos/calculoimc.pdf

a primeira questão da lista, putnam 87, ficou um pouco obscuro para
mim como essa simetria foi utilizada e ainda como sacar em questões do
tipo, que se pode usar simetria.

P.s - onde posso encontrar alguma coisa para ler a respeito,

forte abraço a todos,
Jhonata Emerick Ramos

=
Instruções para entrar na lista, sair da lista e usar a lista em
http://www.mat.puc-rio.br/~nicolau/olimp/obm-l.html
=



=
Instruções para entrar na lista, sair da lista e usar a lista em
http://www.mat.puc-rio.br/~nicolau/olimp/obm-l.html
=


Re: [obm-l] duvida - Vietnam Undergraduate Mathemtics Competition 2001

2007-02-19 Por tôpico Marcio Cohen

Se det(B) nao for 0, entao B admite inversa B^-1, e portanto podemos
escrever A=AB^-1+I. Logo, det(A-I) = det(AB^-1) = det(A)*... = 0 = 1
é autovalor de A (contradição!).

A gente chama uma matriz de nilpotente quando existe um inteiro k tal
que A^k = 0. Verifique que A é nilpotente sse seus autovalores são
todos nulos.

Abraços,
Marcio Cohen

On 2/19/07, Jhonata Ramos [EMAIL PROTECTED] wrote:

Pessoal,
tava olhando essa questão:

Let be given two matrices A, B from M_n(R) such that A^2001 = 0 and AB
= A+B. Show that det(B) = 0.

Source VUMC 2001

Vi uma solução que o cara fala o seguinte:

A^2001=0 = A is nipoltent detA=0
lemma: If X,Y commute, Y nilpotent
then det(X+Y)=detX

Gostaria de saber o que significa, nipoltent (não a tradução :)
e se o lemma dele ali é verdadeiro,

Forte abraço,
Jhonata Emerick Ramos

=
Instruções para entrar na lista, sair da lista e usar a lista em
http://www.mat.puc-rio.br/~nicolau/olimp/obm-l.html
=



=
Instruções para entrar na lista, sair da lista e usar a lista em
http://www.mat.puc-rio.br/~nicolau/olimp/obm-l.html
=


[obm-l] Soluções OBM 2006 (Nível 3)

2006-11-12 Por tôpico Marcio Cohen
 Conforme prometido, eu e o Villard colocamos em www.majorando.com as soluções da OBM 2006. 
 Por enquanto colocamos apenas as soluções do nível 3. 
 Para o nível U, está faltando resolver a 6. Mesmo conversando
com diversos alunos que fizeram a prova ainda não conseguimos resolver
essa questão.
 Se alguém puder enviar a solução, ela será incluída no site no
próximo fim de semana com os devidos créditos (durante a semana é
difícil de arranjarmos tempo).
 Abraços,
 Marcio Cohen


Re: [obm-l] Força de grupo!!

2006-10-31 Por tôpico Marcio Cohen
Prezado Rodolfo,
 Já existe um material bastante extenso de preparação para olimpíada universitária em www.majorando.com
 Em breve, colocaremos também nesse site as soluções da OBM que ocorreu no sábado passado.
 Abraços,
 Marcio CohenOn 10/31/06, Rodolfo Braz [EMAIL PROTECTED] wrote:
Pessoal vamos nos mobilizar pra desenvolver material e lista de
treinamento para o pessoal q se prepara para OBM Universitária. Temos
muitas pessoas q se interessam e podem ajudar!! 
		 
Yahoo! Search 
Música para ver e ouvir: 
You're Beautiful, do James Blunt



Re: [obm-l] Resultado da OBM-2006

2006-10-31 Por tôpico Marcio Cohen
 Prezado Llerer, como um dos responsáveis pela OMERJ, informo que o resultado sairá em novembro. 
 Atenciosamente,
 Marcio CohenOn 10/31/06, Llerer [EMAIL PROTECTED] wrote:




Nelly, e sobre a OMERJ, você sabe alguma coisa ?

- Mensagem Original - 

De: Olimpiada Brasileira de Matematica 
Para: 
Lista de discussao
Data: TerçA, 31 De Outubro De 2006 04:13

Assunto: [obm-l] Resultado da OBM-2006
Caros(as) amigos(as) da lista,O resultado da OBM-2006 somente será publicado durante o mês de dezembro.(provavelmente somente na segunda quinzena do mês).
Por favor aguardem pois estamos em fase de correção.Abraços, Nelly=Instruções para entrar na lista, sair da lista e usar a lista em
http://www.mat.puc-rio.br/~nicolau/olimp/obm-l.html=




Instruções para entrar na lista, sair da lista e usar a lista em
http://www.mat.puc-rio.br/~nicolau/olimp/obm-l.html



Re: [obm-l] (ITA - 90) SISTEMAS LINEARES - questão 17

2006-10-29 Por tôpico Marcio Cohen
Renan,
O fato de se ter D = Dm = 0 para todo m não garante que um sistema indeterminado, como mostra o exemplo:
 x+y+z = 1
 x+y+z = 1
 x+y+z = 2
 no qual se tem D=D1=D2=D3=0 mas o sistema é impossível. 


Ao provar que D=Dm=0 para todo m você pode concluir que o sistema
é indeterminado ou impossível. No caso dessa questão, como (0,0,...,0)
é solução, o sistema é indeterminado.

Abraço,
Marcio Cohen
On 10/28/06, J. Renan [EMAIL PROTECTED] wrote:
Como é um sistema normal, podemos usar a regra de CramerSendo m um número natural qualquer em [1 , n]x_m = Dm/DOnde
Dm denota o determinante da matriz incompleta com os coeficientes de m
trocados pelo termo independente. (e bem... D o determinante da matriz
incompleta)
Bom... já mostraram várias vezes que D = 0, daí x_m = Dm/0Porém Dm também é zero! (Como os coeficientes estão em PA vale usar a soma dos extremos, p. exemplo, pra mostrar que Dm é zero)x_m = 0/0 (indeterminação) para qualquer m no intervalo [1,n]
Em 28/10/06, vinicius aleixo 
[EMAIL PROTECTED] escreveu:

  Considere o sistema linear homogêneo nas incógnitas x_1, x_2, ..., x_n dado por:  
  a_1 .x_1 + (a_1 + 1)x_2 + (a_1 + n - 1)x_n = 0a_2 .x_1 + (a_2 + 1)x_2 + (a_2 + n - 1)x_n = 0...a_n . x_1 + (a_n + 1)x_2 + (a_n + n - 1)x_n = 0  onde a_1, a_2, ..., a_n são número reais dados. Sobre a solução deste sistema podemos afirmar que:
FAz o determinante e veja q ele dara zero(eh mt facil ver isso)  daih,
ele sera ou impossivel ou indeterminado.mas (0,0,...,) eh uma
solucao.logo ele naum eh impossivel e entao eh indeterminado.
 
		 
O Yahoo! está de cara nova. Venha conferir!
-- Um Grande Abraço,Jonas Renan




Re: [obm-l] Transformacao linear

2006-10-29 Por tôpico Marcio Cohen
 Marcelo,
 A resposta é: Depende ddo que foi pedido.
 Como transformações lineares não preservam ângulos, é improvável
que você consiga resolver esse problema através de uma transformação
dessas.
 Abraços,
 Marcio CohenOn 10/28/06, Marcelo Salhab Brogliato [EMAIL PROTECTED] wrote:







Olá,

fiz hj a 2a. fase da OBMU, e fiquei em duvida 
quanto a um passo na minha resolucao da questao da elipse...

se eu tenho uma elipse, aplico uma transformacao 
linear, e obtenho uma circunferencia...
entao, eu provo o q foi pedido para a 
circunferencia... essa prova tambem vale para a 
elipse?

a questao pedia pra provar que, se uma mesa de 
sinuca fosse montada no formado da elipse, e se uma bola colidisse com a borda 
da mesa, ela sairia na posicao simetria em relacao à normal da elipse, entao uma 
bola saindo de A, passando por B e C, e entao retornando a A, passaria novamente 
por B.

obs: a transformacao linear é 
bijetora..
abraços,
Salhab





Re: [obm-l] Sequência

2006-09-17 Por tôpico Marcio Cohen

  Esse problema já caiu numa olimpíada do leste europeu..

  A tabela abaixo mostra que a sequência não pode ter mais que 16
termos (pois somando por linhas a tabela abaixo temos uma soma
positiva, e somando por colunas temos uma soma negativa!).

a1 a2 a3 a4 a5 a6 a7 a8   a9   a10 a11
a2 a3 a4 a5 a6 a7 a8 a9   a10 a11 a12
a3 a4 a5 a6 a7 a8 a9 a10 a11 a12 a13
a4 a5 ...
a5 a6 ...
a6 ...
a7 a8 a9 ...  a17

 Não é fácil construir uma sequência com 16 termos, mas um exemplo é:
 5 5 -13 5 5 -13 5 5 -13 5 5 -13 5 5 -13 5

Abraços,
Marcio Cohen


On 9/15/06, Iuri [EMAIL PROTECTED] wrote:

Numa sequencia finita, temos a soma de 7 termos consecutivos sendo sempre
negativa, e a soma de 11 termos consecutivos sendo sempre positiva. Qual é o
numero máximo de termos dessa sequencia?

Iuri



=
Instruções para entrar na lista, sair da lista e usar a lista em
http://www.mat.puc-rio.br/~nicolau/olimp/obm-l.html
=


[obm-l] IMO 2006 Eslovênia

2006-07-13 Por tôpico Marcio Cohen

   Prezados participantes da lista,

   A IMO 2006 já está disponível, inclusive com as soluções oficiais. Eu as 
coloquei em www.majorando.com , mas também é possível encontrá-las no site 
oficial dessa IMO.


   Esse site foi criado por mim e pelo Rodrigo Villard (ele já foi um 
participante ativo dessa lista).


   Nele você encontrará detalhes de um livro que acabamos de escrever e 
será lançado em agosto com tópicos teóricos e soluções das provas de 
matemática do IME dos últimos 15 anos.


   Encontrará também diversos artigos escritos por nós relacionados a 
olimpíadas de matemática, incluindo artigos de preparação para o vestibular 
do IME, olimpíadas de ensino médio (níveis intermediário e avançado) e 
olimpíadas universitárias (nível avançado).


Abraços,
Marcio Cohen


=
Instruções para entrar na lista, sair da lista e usar a lista em
http://www.mat.puc-rio.br/~nicolau/olimp/obm-l.html
=


Re: [obm-l] LIMITES

2006-05-22 Por tôpico Marcio Cohen

 É verdade, obrigado pela correção!
 Marcio

- Original Message - 
From: Ricardo Bittencourt [EMAIL PROTECTED]

To: obm-l@mat.puc-rio.br
Sent: Monday, May 22, 2006 1:12 AM
Subject: Re: [obm-l] LIMITES



Marcio Cohen wrote:


 Oi Marcelo.
 Você pode multiplicar S por (1-1/2)/(1-1/2) e concluir que S não só 
converge, mas tem forma fechada simples.
 Usando que (a+b)(a-b) = a^2 - b^2 repetidas vezes (ou por indução), 
S(n) =  2*(1 - 1/2^(n+1)), logo S tende a 2.


Na verdade S(3)=3*5*9/(2*4*8)=135/64=2.109375  2, e como
S(n)S(n+1) com certeza ela converge pra algo maior que 2.

O erro no seu raciocínio é que você gera termos
da forma (1+1/2^2^n), e não (1+1/2^n) como você gostaria.

Numericamente, esse limite converge para aproximadamente 2.384231.


Ricardo Bittencourt   http://www.mundobizarro.tk
[EMAIL PROTECTED]  kimitatino kitiwa subete CATS ga itadaita
-- União contra o forward - crie suas proprias piadas --
=
Instruções para entrar na lista, sair da lista e usar a lista em
http://www.mat.puc-rio.br/~nicolau/olimp/obm-l.html
=



=
Instruções para entrar na lista, sair da lista e usar a lista em
http://www.mat.puc-rio.br/~nicolau/olimp/obm-l.html
=


Re: [obm-l] LIMITES

2006-05-21 Por tôpico Marcio Cohen



Para ser mais preciso (e chato), 
 -1/|x| = sen(a)/x = 
1/|x|

  - Original Message - 
  From: 
  Marcelo Salhab 
  Brogliato 
  To: obm-l@mat.puc-rio.br 
  Sent: Sunday, May 21, 2006 9:10 PM
  Subject: Re: [obm-l] LIMITES
  
  Olá,
  
  pq -1 = sen(a) = 1.. para qualquer 
  a...
  dividindo por x, temos:
  
  -1/x = sen(a)/x = 1/x
  
  abracos,
  Salhab
  
- Original Message - 
From: 
Klaus Ferraz 
To: obm-l@mat.puc-rio.br 
Sent: Sunday, May 21, 2006 6:43 
PM
Subject: Re: [obm-l] LIMITES
Porque -1/x = sen(x^1000)/x = 1/x é 
verdade??Marcelo Salhab Brogliato [EMAIL PROTECTED] escreveu: 

  
  

  Olá
  
  2)
  -1/x = sen(x^1000)/x = 
  1/x
  
  qdo x - +inf.. -1/x e 1/x tendem para 0.. 
  pelo teorema do confronto (sanduiche), o limite de sen(x^1000)/x - 0 
  quando x- 0.
  
  abraços,
  Salhab
  
- Original Message - 
From: 
Klaus Ferraz 
To: obm-l@mat.puc-rio.br 
Sent: Sunday, May 21, 2006 10:37 
AM
Subject: [obm-l] LIMITES

1)Determine lim(n-+inf) 
(1+1/2)*(1+1/2^2)*(1+1/2^3)*...*(1+1/2^n).
2)Determine lim(x--+inf) sen(x^1000)/x

Grato.


Yahoo! 
Messenger com voz - Instale agora e faça ligações de graça. 
  


Abra 
sua conta no Yahoo! Mail - 1GB de espaço, alertas de e-mail no celular e 
anti-spam realmente eficaz. 


Re: [obm-l] LIMITES

2006-05-21 Por tôpico Marcio Cohen



Oi Marcelo. 
Você pode multiplicar S por (1-1/2)/(1-1/2) e 
concluir que S não só converge, mas temforma fechada simples. 

Usando que (a+b)(a-b) = a^2 - b^2 repetidas 
vezes (ou por indução),S(n) 
=2*(1 - 1/2^(n+1)), logo S tende a 2. 






  - Original Message - 
  From: 
  Marcelo Salhab 
  Brogliato 
  To: obm-l@mat.puc-rio.br 
  Sent: Monday, May 22, 2006 12:32 AM
  Subject: Re: [obm-l] LIMITES
  
  Olá,
  consegui algumas coisas na 1.. mas ainda nao 
  cheguei a uma resposta..
  
  1)
  Seja S = (1 + 1/2)(1+1/2^2)(1+1/2^3)...(1+1/2^n), 
  temos que:
  
  lnS = ln(1+1/2) + ln(1+1/2^2) + ln(1+1/2^3) + ... 
  + ln(1+1/2^n)
  
  é fácil mostrar que o somátorio a direita, quando 
  n-inf, converge. Pois aplicando o teste da raiz obtemos 1/2  
  1.
  
  sabemos que ln(1+x) = x .. 
  x=0
  
  assim: ln(1+1/2^k) = 1/2^k
  
  logo, Sum(k=1..n, ln(1+1/2^k))  1/2 * (1 - 
  1/2^n)/1/2= 1 - 1/2^n
  
  qdo n-inf, temos: Sum(k=1..inf, 
  ln(1+1/2^k))= 1
  
  assim, lnS = 1, qdo n-inf
  logo: S = e, qdo n-inf
  
  bom, talvez conseguindo mostrar que S = e... 
  ou entao utilizando outra ideia pra concluir a questao.
  
  abraços,
  Salhab
  
  
- Original Message - 
From: 
Klaus Ferraz 
To: obm-l@mat.puc-rio.br 
Sent: Sunday, May 21, 2006 6:45 
PM
Subject: Re: [obm-l] LIMITES

Ola Carlos,
 A questao 1 estah ok. eh isso mesmo. tem algumas opcoes:
a)1/2 b)1 c)3/2 d)2 e)4Carlos Victor [EMAIL PROTECTED] 
escreveu:
Olá 
  ,Para o segundo limite temos :lim(x--+inf) 
  sen(x^1000)/x = lim( 1/x.sen(x^1000) , como 
  sendo uma função infitesima multiplicada por 
  um limitada ; ou seja a resposta é 
  zero .Tem certeza que a 
  questão (1) esta correta ?[]´s 
  Carlos VictorAt 10:37 21/5/2006, Klaus Ferraz wrote:
  1)Determine lim(n-+inf) 
(1+1/2)*(1+1/2^2)*(1+1/2^3)*...*(1+1/2^n).2)Determine 
lim(x--+inf) sen(x^1000)/xGrato.Yahoo! 
Messenger com voz - Instale agora e faça ligações de graça. 
  


Abra 
sua conta no Yahoo! Mail - 1GB de espaço, alertas de e-mail no celular e 
anti-spam realmente eficaz. 


Re: [obm-l] Problema de geometria plana

2006-04-26 Por tôpico Marcio M Rocha

[EMAIL PROTECTED] escreveu:


Srs,

O problema abaixo é o de número 55 do livro matematica para o
vestibular da UFMG
(geometria plana) do Prof Christiano Sena.

(sem acentos)
Num triangulo ABC, AB =8 cm e AC = 10cm. Pelo incentro do triangulo,
traca-se uma reta paralela
a BC, que intercepta AB em M e AC em N. O perimetro do triangulo AMN eh:
a) 16 b) 17 c) 18 d) 19 e) 20.

alguem sabe sua solução? o gabarito diz que é 20.

at

sarmento




Aqui na Oi Internet você ganha ou ganha. Além de acesso grátis com qualidade, 
ganha contas ilimitadas de email com 1 giga cada uma. Ganha 60 mega para hospedar 
sua página pessoal. Ganha flog, suporte grátis e muito mais. Baixe grátis o 
Discador em http://www.oi.com.br/discador e comece a ganhar.


Agora, se o seu negócio é voar na internet sem pagar uma fortuna, assine 
Oi Internet banda larga por apenas R$ 9,90. Clique em 
http://www.oi.com.br/bandalarga e aproveite essa bocada!



=
Instruções para entrar na lista, sair da lista e usar a lista em
http://www.mat.puc-rio.br/~nicolau/olimp/obm-l.html
=


 



Boa noite, Sarmento.

Seja I o incentro do triângulo. Sabe-se que med(MBI)=med(IBC) e que 
med(NCI)=med(ICB). Por outro lado, sendo MN paralelo a BC, tem-se que 
med(MIB)=med(IBC)=med(MBI) e med(NIC)=med(ICB)=med(NCI). Daí: MB = MI e 
NC = NI.


O perímetro de AMN é:

AM + MN + NA = AM + MI + IN + NA = AM + MB + NC + NA = AB + AC = 18.

Se algo estiver errado, leve em conta o horário.

Abraços,

Márcio.

=
Instruções para entrar na lista, sair da lista e usar a lista em
http://www.mat.puc-rio.br/~nicolau/olimp/obm-l.html
=


Re: [obm-l] Polinomio

2006-04-04 Por tôpico Marcio M Rocha

Klaus Ferraz escreveu:

Mostre que se a,b,c sao inteiros impares, a equacao ax^2+bx+c nao tem 
raiz racional.



Novidade no Yahoo! Mail: receba alertas de novas mensagens no seu 
celular. Registre seu aparelho agora! 
http://us.rd.yahoo.com/mail/br/tagline/mobile_alerts/*http://br.mobile.yahoo.com/mailalertas/ 





No virus found in this incoming message.
Checked by AVG Free Edition.
Version: 7.1.385 / Virus Database: 268.3.5/301 - Release Date: 4/4/2006
 

Leia o artigo de Eduardo Wagner, Paridade, que pode ser encontrado em 
http://www.obm.org.br/frameset-eureka.htm.


Márcio.
=
Instruções para entrar na lista, sair da lista e usar a lista em
http://www.mat.puc-rio.br/~nicolau/olimp/obm-l.html
=


Re: [obm-l] Geometria espacial

2006-03-22 Por tôpico Marcio Cohen

 Ponciano, sua solução está completa e elegante.

- Original Message - 
From: Ronaldo Luiz Alonso [EMAIL PROTECTED]

To: obm-l@mat.puc-rio.br
Sent: Tuesday, March 21, 2006 4:54 PM
Subject: Re: [obm-l] Geometria espacial



Tudo bem...
Mas precisa justificar ...  Será que esse arranjo de pontos
maximiza o número de pontos que podem ser colocados dentro do cubo?
H não tenho tanta certeza...



- Original Message - 
From: João Gilberto Ponciano Pereira [EMAIL PROTECTED]

To: obm-l@mat.puc-rio.br
Sent: Tuesday, March 21, 2006 3:59 PM
Subject: RE: [obm-l] Geometria espacial



Estava pensando numa forma mais simples...

Dividir o cubo unitário em 125 cubinhos de lado 1/5

Por casa dos pombos, ao menos um desses cubinhos possui 4 pontos em seu 
interior. E como uma esfera de raio 1/5 contém um cubo de raio 1/5


-Original Message-
From: [EMAIL PROTECTED] [mailto:[EMAIL PROTECTED]
Behalf Of Ronaldo Luiz Alonso
Sent: Tuesday, March 21, 2006 3:22 PM
To: obm-l@mat.puc-rio.br
Subject: Re: [obm-l] Geometria espacial


Esse problema foi resolvido em uma revista do professor de matemática.
  Vou apenas esboçar como faz ...
  Parece que não mas esse é um problema de química.
   Troque cubo unitário por célula unitária e pontos por átomos
   Quem não sober o que é cela unitária digite célula unitária no 
Google.


   Eu acredito que a melhor situação seria aquela em que os pontos estão 
em

em um reticulado (lattice em inglês) uniformemente espaçado.
 Neste caso temos que colocar o maior número de pontos
possíveis dentro deste  reticulado.
  O reticulado então tem que ser um reticulado de Bravais.
Existem 7 reticulados de Bravais que preenchem o espaço.

http://pt.wikipedia.org/wiki/Rede_de_Bravais

Para todos esses 7 reticulados, no caso do problema
existem pelo menos 4 pontos dentre os 400 que fazem pate dos vértices
que estão no interior de uma  esfera de raio 1/5.

Quem não concordar com isso, diga agora ou cale-se para sempre :)

- Original Message - 
From: Dymitri Cardoso Leão [EMAIL PROTECTED]

To: obm-l@mat.puc-rio.br
Sent: Tuesday, March 21, 2006 2:25 PM
Subject: [obm-l] Geometria espacial



* Colocamos 400 pontos, distintos dois a dois, no interior de um cubo
unitário. Prove que,
entre os 400 pontos, existem pelo menos 4 que estão no interior de uma
esfera de raio 1/5.

Não tenho a menor noçao de como fazer isto, alguém poderia por favor
resolver?


=
Instruções para entrar na lista, sair da lista e usar a lista em
http://www.mat.puc-rio.br/~nicolau/olimp/obm-l.html
=


[obm-l] Re: [obm-l] polinômios de Taylor

2006-03-19 Por tôpico Marcio Cohen



  f(x) = -1/(1-x) = 
-(1+x+x^2+x^3+x^4+x^5+...).
 Logo, o polinômio de taylor de 
ordem 2 em torno de x=0 é (-x^2-x-1). Ficou faltando um sinal de menos no seu 
coeficiente líder.

 Abraços,
 Marcio


  - Original Message - 
  From: 
  Tiago Machado 
  
  To: obm-l@mat.puc-rio.br 
  Sent: Sunday, March 19, 2006 7:45 
PM
  Subject: [obm-l] polinômios de 
  Taylor
  Alguem pode me confirmar se o polinômio de Taylor (de ordem 2) 
  para a função f(x) = 1/(x - 1) no ponto x = 0 é x² - x - 
  1?Obrigado.


[obm-l] Re: [obm-l] Re: [obm-l] Fw: congruência

2006-03-04 Por tôpico Marcio Cohen



 Como 23 eh primo, 10^22 = 1 (mod 23), e 
como10^2 = 8 e 10^11 = 10*(10^2)^5= 10*8^5 = 10*16 != 1 (mod 
23),22 eh o menor numero com essa 
propriedade. 
 Logo, 10^a = 10^b (mod 23) se e somente se a 
= b (mod 22).
 Como 10^2 = 8 (mod 23), a resposta é que os 
valores de k para os quais temos 10^k = 8 (mod 23) são exatamente os inteiros 
positivos que deixam resto 2 na divisão por 22 (2, 24, 46, ...)

 Abraços,
 Marcio



  - Original Message - 
  From: 
  Marcelo Salhab 
  Brogliato 
  To: obm-l@mat.puc-rio.br 
  Sent: Saturday, March 04, 2006 7:06 
  PM
  Subject: [obm-l] Re: [obm-l] Fw: 
  congruência
  
  Olá,
  
  vc quer saber para quais valores de k 
  temos:
  10^k = 8 (mod 23), certo?
  
  bom, temos que:
  100 = 8 (mod 23)
  10^(2n) = 8^n (mod 23)
  isso é, para k par temos que a unica solucao é 
  k=2 (n=1).
  
  ainda nao consegui extender essa solucao para k 
  impar.. estou tentando!
  
  PS: sei mto pouco sobre congruencia, talvez minha 
  solucao esteja errada
  
  abraços,
  Salhab
  
  
  
- Original Message - 
From: 
Leo 

To: obm-l@mat.puc-rio.br 
Sent: Saturday, March 04, 2006 12:16 
AM
Subject: [obm-l] Fw: congruência


- Original Message - 
From: Leo 
To: obm-l@mat.puc-rio.br 
Sent: Friday, March 03, 2006 8:11 PM
Subject: congruência

Como resolver a seguinte 
congruência
10^k cong 8 (mod 23) ... pra k=2 eh verdadeira 
mas como achar o caso 
geral???


Re: [obm-l] Re:[obm-l] Fatoração

2006-02-22 Por tôpico Marcio M Rocha

Salhab [ k4ss ] escreveu:



 
(a+b+c)^4 = 1

*fatorando*.. temos:
a^4 + b^4 + c^4 + 4 [(ab)^2 + (bc)^2 + (ac)^2 + 2abc] = 1
a^4 + b^4 + c^4 + 4 * 1/4 = 1
a^4 + b^4 + c^4 = 0
 

 
 

Sem querer ser chato, gostaria de fazer uma pequeníssima correção nas 
palavras: no caso, o correto é *expandindo*, e não fatorando.


Márcio.


=
Instruções para entrar na lista, sair da lista e usar a lista em
http://www.mat.puc-rio.br/~nicolau/olimp/obm-l.html
=


Re: [obm-l] altura_triangulo

2006-02-15 Por tôpico Marcio M Rocha

elton francisco ferreira escreveu:


ola pessoal da lista!
como resolver a questao que se segue?

Determine a altura nao relativa a base de um triangulo
isosceles de lados 10m, 10m e 12 m.








___ 
Yahoo! doce lar. Faça do Yahoo! sua homepage. 
http://br.yahoo.com/homepageset.html 


=
Instruções para entrar na lista, sair da lista e usar a lista em
http://www.mat.puc-rio.br/~nicolau/olimp/obm-l.html
=


 


1) Encontre a altura relativa à base;
2) Calcule a área do triângulo;
3) Sendo *h* a altura procurada e sendo a base (agora) igual a 10m, use 
a informação de 2) para calcular *h*.


Márcio.
=
Instruções para entrar na lista, sair da lista e usar a lista em
http://www.mat.puc-rio.br/~nicolau/olimp/obm-l.html
=


Re: [obm-l] inducao

2005-12-17 Por tôpico Marcio Cohen



A 2a é maior que a 1a ué... 

  - Original Message - 
  From: 
  Klaus 
  Ferraz 
  To: obm-l@mat.puc-rio.br 
  Sent: Saturday, December 17, 2005 10:16 
  AM
  Subject: [obm-l] inducao
  
  Mostre usando inducao que para todo natural 
  n:1/n+1 + 1/n+2 + ...+1/2n = 1/2
  
  Mostre que para todo natural n: 1/n + 1/n+1 + 1/n+2 + ...+1/2n = 
  1/2
  
  a primeira dá por inducao só q nao consegui. a segunda não dá. 
  
  
  Yahoo! doce lar. Faça 
  do Yahoo! sua homepage.


Re: [obm-l] Procuro um livro

2005-11-29 Por tôpico marcio aparecido
Olá José pode me informar onde você encontro esse exemplar ruim, se
você não for querer presentear a pessoa com ele!!

=
Instruções para entrar na lista, sair da lista e usar a lista em
http://www.mat.puc-rio.br/~nicolau/olimp/obm-l.html
=


[obm-l] Ajuda Vestibular ITA

2005-11-24 Por tôpico marcio aparecido
Olá pessoal
To precisando da ajuda de vcs. Vou prestar vestibular para o ITA no
próximo ano, e estou
precisando de materiais legais mesmo para estudar. Se vcs conhecem algum
site/livros/métodos de estudo bom mesmo com esses materiais
respondam-me por favor. Não precisa ser
apenas de Matématica pode ser Física, Química, Português, Inglês. Qualquer
ajuda será válida. Valeu Galera.

=
Instruções para entrar na lista, sair da lista e usar a lista em
http://www.mat.puc-rio.br/~nicolau/olimp/obm-l.html
=


Re: [obm-l] RE: [obm-l] Noções de Matemática

2005-11-23 Por tôpico marcio aparecido
Se você puder me passar o endereço eu agradeço Felipe!!

=
Instruções para entrar na lista, sair da lista e usar a lista em
http://www.mat.puc-rio.br/~nicolau/olimp/obm-l.html
=


[obm-l] Noções de Matemática

2005-11-22 Por tôpico marcio aparecido
alguem aqui da comunidade sabe onde eu posso encontrar para compra a
coleção Noções de Matemática (Aref / Nilton Lapa) para compra.
E também a indicação de sebos bons para compra de livrvos de exatas!!!
Abraços

=
Instruções para entrar na lista, sair da lista e usar a lista em
http://www.mat.puc-rio.br/~nicolau/olimp/obm-l.html
=


[obm-l] Proposição

2005-11-21 Por tôpico marcio aparecido
Alguem pode me ajudar a entender o conceito de proposição!!

=
Instruções para entrar na lista, sair da lista e usar a lista em
http://www.mat.puc-rio.br/~nicolau/olimp/obm-l.html
=


[obm-l] Inteiros

2005-11-19 Por tôpico marcio aparecido
Se p natural maior que 1 não é divisivel nem por 2 nem por 3, então
p^2-1 é divisilvel por:
a)18  b)24  c)36  d)9  e)27

=
Instruções para entrar na lista, sair da lista e usar a lista em
http://www.mat.puc-rio.br/~nicolau/olimp/obm-l.html
=


[obm-l] Algebra inteiros

2005-11-16 Por tôpico marcio aparecido
Seja m um número ímpar, prove que m^2 -1 é divisil por 8.

=
Instruções para entrar na lista, sair da lista e usar a lista em
http://www.mat.puc-rio.br/~nicolau/olimp/obm-l.html
=


Re: [obm-l] trigonometria

2005-11-16 Por tôpico Marcio Cohen



Pondo(senx)^2 =1/2+t, (cosx)^2=1/2-t, 
-1/2=t=1/2
 y = (1/2+t)^3 + (1/2-t)^3 = 1/4 + 3t^2 
tem mínimo em t=0 (y=1/4) e máximo em t=+-1/2 (y=1)
Observe que y=1/4 para x=pi/4, 3pi/4, 5pi/4, ... 
logo o período é maior ou igual a pi/2.
Por outro lado, trocar x por x+pi/2 não muda o 
valor de y, logo o período é exatamente pi/2.

  - Original Message - 
  From: 
  Guilherme Neves 
  To: obm-l@mat.puc-rio.br 
  Sent: Wednesday, November 16, 2005 9:46 
  PM
  Subject: [obm-l] trigonometria
  
  
  Achar os valores maximo e minimo e o periodo da função 
  y=(sen(x))^6 + (cos(x))^6 
  fiz de uma maneira mt trabalhosa que se ninguem tiver feito 
  igualmente eu coloco aqui. 
  = 
  Instruções para entrar na lista, sair da lista e usar a lista em http://www.mat.puc-rio.br/~nicolau/olimp/obm-l.html 
  = 



Re: [obm-l] Re:[Spam] [obm-l] questões!!

2005-11-14 Por tôpico marcio aparecido
questões para praticar, pegar velocidade na resoluções.
pode ter questões de todos os níveis (fácil, media e dificil).


Re: [obm-l] Re:[obm-l] questões!!

2005-11-14 Por tôpico marcio aparecido
Obrigado pelas questões Paulo.
Alguem pode me ajudar com livros também.
Abraços Márcio.

=
Instruções para entrar na lista, sair da lista e usar a lista em
http://www.mat.puc-rio.br/~nicolau/olimp/obm-l.html
=


Re: [obm-l] polinomio

2005-11-14 Por tôpico Marcio Cohen



P(x) = x eh a unica solução (demo: P(x)-x se anula 
em todos os pontos da seq. crescente definida por a1=1, 
a(n+1)=a(n)^2+1, n =1 e portanto é 
identicamente nulo)

  - Original Message - 
  From: 
  Danilo Nascimento 
  To: obm-l@mat.puc-rio.br 
  Sent: Monday, November 14, 2005 8:29 
  PM
  Subject: [obm-l] polinomio
  
  Determine todos os polinomios P(x) tais que P(x^2+1) = (P(x))^2+1 para 
  todo x real.
  
  alguem se habilita?
  
  
  Yahoo! Acesso Grátis: Internet rápida e grátis.Instale 
  o discador agora!


[obm-l] questões!!

2005-11-13 Por tôpico marcio aparecido
alguem sabe onde eu posso encontrar questões boas, ótimas de 5º a 8º série ??
e também livros ??

=
Instruções para entrar na lista, sair da lista e usar a lista em
http://www.mat.puc-rio.br/~nicolau/olimp/obm-l.html
=


[obm-l] Proposição ??

2005-11-12 Por tôpico marcio aparecido
alguem pode me da uma ajuda com o conceito de proposição ??

=
Instruções para entrar na lista, sair da lista e usar a lista em
http://www.mat.puc-rio.br/~nicolau/olimp/obm-l.html
=


[obm-l] Problemas de Sistema de Númeração

2005-11-12 Por tôpico marcio aparecido
1) Um número tem três alagarimos, sendo que o número formado pelos
dois algarismo a direita é o quadrado dos algarismo das centenas. A
diferença entre o algarismo das centens e os das dezenas é o
algarismos das unidades. Qual é esse número ?

2) Um número par tem dois algarismo, cuja a soma é 15 e cuja a
diferença em módulo é 3. Qual o número ??

=
Instruções para entrar na lista, sair da lista e usar a lista em
http://www.mat.puc-rio.br/~nicolau/olimp/obm-l.html
=


[obm-l] Tautologia

2005-11-05 Por tôpico marcio aparecido
Olá galera tô com uma dúvida bem básica:
O que é uma Tautologia e para que ele serve, sua aplicação
Abraços.

=
Instruções para entrar na lista, sair da lista e usar a lista em
http://www.mat.puc-rio.br/~nicolau/olimp/obm-l.html
=


[obm-l] lidski

2005-10-28 Por tôpico marcio aparecido
Oieee galera
queria pergintar se alguem aqui da lista tem o lidski, e em que línguas
ele foi publicado aqui no brasil ??

=
Instruções para entrar na lista, sair da lista e usar a lista em
http://www.mat.puc-rio.br/~nicolau/olimp/obm-l.html
=


Re: [obm-l] equacao

2005-10-26 Por tôpico Marcio Cohen



 Duas soluções para essa questão, bem como as 
soluções de todas as questões da prova de matemática do IME desse ano podem ser 
encontradas por exemplo no site do Ponto de Ensino (onde eu 
trabalho):
 www.pensi.com.br

Uma solução possível é: Como k eh primo, xy 
multiplo de k = x ou y multiplo de k.
Se x=ak, a inteiro, temos substituindo na 
equacao que y=ak/(a-1). Como y eh inteiro e mdc(a,a-1)=1, deve-se ter a-1 
dividindo k. Sendo k primo, isso dá a-1 em {-k,-1,k,1} que dá a em {1-k, 0, 1+k, 
2} e substituindo em x=ak e y=ak/(a-1) vc acha 4 soluções. Trocando x com y 
(i.e, fazendo y=ak) vc acha mais duas.

 É interessante notar que essa questão 
já tinha aparecido antes na olimpíada de matemática do Estado do Rio de Janeiro 
de 1998 (por acaso foi uma prova que eu fiz como aluno, por isso lembrei 
:)).
 
 Abraços,
 Marcio

  - Original Message - 
  From: 
  Danilo Nascimento 
  To: obm-l@mat.puc-rio.br 
  Sent: Wednesday, October 26, 2005 9:28 
  AM
  Subject: [obm-l] equacao
  
  Determine o conjunto solucao d (x+y)k = xy sendo x e y inteiros 
  positivose k um numero primo
  
  
  Promoção Yahoo! Acesso Grátis: a cada hora navegada vocêacumula cupons e 
  concorre a mais de 500 prêmios! Participe!


Re: Re:[obm-l] equacao

2005-10-26 Por tôpico Marcio Cohen



 Mesmo assim, ainda temos as soluções: 
(k^2+k, k+1) e (k-k^2, k-1) e suas simétricas.

  - Original Message - 
  From: 
  claudio.buffara 
  To: obm-l 
  Sent: Wednesday, October 26, 2005 1:14 
  PM
  Subject: Re:[obm-l] equacao
  
  Eu supuz que k é um primo fixo dado.
  
  
  


  De:
  [EMAIL PROTECTED]
  
  


  Para:
  "obm-l" obm-l@mat.puc-rio.br
  
  


  Cópia:
  
  
  


  Data:
  Wed, 26 Oct 2005 
12:20:17 -0200 (BRST)
  
  


  Assunto:
  Re:[obm-l] 
equacao
   
   Na verdare, por tentativa (e muitos erros)
   e' possivel tambem outras solucoes:
   
   zk - zw = -wk
   = z = -wk/(k-w)
   Logo, se k = (w+1) entao z = -w(w+1)
   
   Por simetria, se k = (z+1) entao w = -z(z+1)
   
   Abraco,
   sergio
   
   On Wed, 26 Oct 2005, claudio.buffara wrote:
   
Seja d = mdc(x,y). Então x = dz e y = dw, com mdc(z,w) = 
  1.

A equação fica (z + w)k = dzw.

k não pode dividir z pois z = km ==
(km + w)k = dkmw ==
km + w = dmw ==
w = m(dw - k) ==
m divide w ==
contradição, pois z (e portanto m) é primo com w

Da mesma forma, vemos que k não pode dividir w.

Logo, k divide d ==
d = kn ==
(z + w)k = knzw ==
z + w = nzw ==
1/w + 1/z = n = inteiro positivo

Como z e w são inteiros positivos, 1/z + 1/w = 2.

Se z = w = 1, então x = y = d == 2dk = d^2 == d = 2k 
  ==
uma solução é (2k,2k).

Se z  1 ou w  1, então 1/z + 1/w = n = 1 ==
z = w = 2 e d = k ==
de novo obtemos a solução (2k,2k).

Logo, a única solução é (2k,2k).


De:[EMAIL PROTECTED]

Para:obm-l@mat.puc-rio.br

Cópia:

Data:Wed, 26 Oct 2005 11:28:09 + (GMT)

Assunto:[obm-l] equacao

 Determine o conjunto solucao d (x+y)k = xy sendo x e y 
  inteiros positivos e k um numero primo


Promoção Yahoo! Acesso Grátis: a cada hora navegada você
acumula cupons e concorre a mais de 500 prêmios! 
  Participe!

   
   
  =
   Instruções para entrar na lista, sair da lista e usar a lista 
  em
   http://www.mat.puc-rio.br/~nicolau/olimp/obm-l.html
   
  =
   


[obm-l] lidski alguem ??

2005-10-21 Por tôpico marcio aparecido
Oieee galera
queria pergintar se alguem aqui da lista tem o lidski e em que línguas
ele foi publicado aqui no brasil ??
Abraços Márcio!!

=
Instruções para entrar na lista, sair da lista e usar a lista em
http://www.mat.puc-rio.br/~nicolau/olimp/obm-l.html
=


[obm-l] Ensino de matematica

2005-10-17 Por tôpico Marcio M Rocha
Gostaria de fazer uma pergunta ao pessoal da lista que lida com ensino, 
principalmente (mas não exclusivamente) a nivel fundamental e médio: por 
que o Brasil tem um desempenho tão bom nas competições internacionais de 
matemática mas, ao mesmo tempo, é tão mal avaliado no, vamos dizer, 
ensino normal? Pergunto isso porque há países que têm qualidade tanto na 
matemática olímpica quanto na matemática escolar.

Qual a sua opinião, professor Nicolau?
Desculpem se o assunto foge ao tema principal.

[]s,

Márcio.
=
Instruções para entrar na lista, sair da lista e usar a lista em
http://www.mat.puc-rio.br/~nicolau/olimp/obm-l.html
=


Re: [obm-l] Aritmética Progressiva

2005-10-08 Por tôpico Marcio M Rocha

Ilídio Leite escreveu:


olá...
 
alguém conhece o livro Aritmética Progressiva, de Antônio Trajano?

achei um exemplar num sebo mas estava encapado, não pude folheá-lo.
 
 
abraços a todos,

Ilídio Leite


Caro Ilídio,

Eu tinha um exemplar deste livro. Fiquei muito empolgado quando comprei 
porque o Elon (que ele me desculpe a intimidade) fala sempre muito bem 
deste livro. Bem, entrei em contato com o próprio Elon falando sobre o 
livro e coisa e tal e aí veio o balde de água fria: minha edição não era 
a que ele usara, ou seja, o exemplar que estava comigo já havia sofrido 
alterações e reduções. Havia comprado gato por lebre.


Se você conseguir uma permissão para dar uma olhada no interior do 
livro, procure se consta a definição de proporcionalidade. Se constar, 
compre o livro (e depois vou tentar uma cópia contigo!), caso contrário, 
não creio que o investimento valha a pena, mas aí é contigo.


Abraços,

Márcio.
=
Instruções para entrar na lista, sair da lista e usar a lista em
http://www.mat.puc-rio.br/~nicolau/olimp/obm-l.html
=


Re: [obm-l] Fw: Probabilidade

2005-10-05 Por tôpico Marcio Cohen



Sim. A questão é da olimpíada estadual de 
matemática de 2005, mas o enunciado não é exatamente assim (embora o sentido 
seja esse).

  - Original Message - 
  From: 
  fgb1 
  To: obm-l@mat.puc-rio.br 
  Sent: Wednesday, October 05, 2005 9:45 
  PM
  Subject: [obm-l] Fw: Probabilidade
  
  
  - Original Message - 
  From: fgb1 
  To: obm-l@mat.puc-rio.br 
  Sent: Saturday, February 19, 2005 11:59 PM
  Subject: Probabilidade
  
  Uma roleta circular foi dividia em 6 setores de 
  mesma área. Em 3 desses setores estava escrito: ganha o carro. Em 2 desses 
  setores estava escrito: Perde o carro e no outro : jogue novamente. Qual é a 
  probabilidade de um jogador ganhar o carro?
  
  O aluno me disse que era de uma olimpíada 
  recente. Alguém, por acaso,reconhece a Olímpíada?
  


Re: [obm-l] Ajuda - Proporção

2005-10-01 Por tôpico Marcio M Rocha

admath escreveu:


Olá
 
Já li diversas teorias sobre proporcionalidade só que não consigo 
entender estes dois problemas de maneira alguma. Alguém pode me 
explicar de uma maneira bem didática?
 
1) Dividindo 70 em partes proporcionais a 2, 3 e 5, a soma entre a 
menor e a maior parte é quanto?
 
2) A proporção entre as medalhas de ouro, prata e bronze de um 
atleta é 3 : 4 : 7, respectivamente. Quantas medalhas de ouro, prata e 
bronze espera-se que esse atleta obtenha em 70 jogos, se essa 
proporção se mantiver e ele conquistar medalhas em todos os jogos?
 
-Posso falar que o método da regra de 3 é o mesmo processo quando 
lidamos com grandezas proporcionais?
 
obrigado.


1) Complementando a solução do Jefferson: Quando o problema menciona 
partes proporcionais', está implícito que são partes DIRETAMENTE 
proporcionais. Quando ele diz que vai dividir 70 em partes proporcionais 
a 2, 3 e 5, isso significa que há um valor constante (chamado constante 
de proporcionalidade) e que a primeira parte vale o dobro, a segunda o 
triplo e a terceira o quíntuplo dessa constante.


Esse problema também pode ser resolvido usando um método conhecido como 
falsa posição. A idéia é a seguinte:


Se as partes valessem 2, 3 e 5, a soma valeria 10. Como a soma das 
partes é 70, que é 7 vezes 10, cada parte deve ser, também, multiplicada 
por 7. Logo, cada parte vale 14, 21 e 35.


2) Usando a idéia da falsa posição:

Para ganhar 3 medalhas de ouro, 4 de prata e 7 de bronze ele deve 
disputar 3 + 4 + 7 = 14 jogos.

Se ele disputar 70 jogos (= 5 x 14) ele ganhará:

   5 x 3 = 15 medalhas de ouro.

   5 x 4 = 20 medalhas de prata.

   5 x 7 = 35 medalhas de bronze.

No primeiro problema a constante de proporcionalidade é 10, e no segundo, 5.

Espero que, com a solução do Jefferson e esses breves comentários, tudo 
tenha ficado mais claro para você.


[]s,

Márcio.
=
Instruções para entrar na lista, sair da lista e usar a lista em
http://www.mat.puc-rio.br/~nicolau/olimp/obm-l.html
=


Re: [obm-l] Livros da MIR

2005-09-05 Por tôpico Marcio
A editora Atual lançou alguns títulos da MIR na série Aprendendo e  
ensinando.
Vejam em http://www.atualeditora.com.br/campanha2002/frame_segmento.htm,  
clicando em Ensino Superior e, logo depois, em Série russa.


[]s,

Márcio.

On Sun, 04 Sep 2005 19:40:30 -0700, Jefferson Franca  
[EMAIL PROTECTED] wrote:



Mesmo assim vc ajudou muito dando esse endereço.
Valeu

Marcio [EMAIL PROTECTED] escreveu:
De fato, passei o endereço errado. O correto (testei antes de escrever) é
realmente www.livifusp.com.br.

[]s,

Márcio.

On Sun, 04 Sep 2005 14:06:12 -0700, Bernardo Freitas Paulo da Costa
wrote:


Cara, o site mudou de nome (tinha uma pagina velha que ainda tinha
isso, a unica no Google, e que disse que isso era a Livraria de Fisica
da Usp), agora tem um i a mais: http://www.livifusp.com.br/. Espero
que seja este mesmo.

Abraços,








--
Using Opera's revolutionary e-mail client: http://www.opera.com/mail/
=
Instruções para entrar na lista, sair da lista e usar a lista em
http://www.mat.puc-rio.br/~nicolau/olimp/obm-l.html
=


Re: [obm-l] Livros da MIR

2005-09-04 Por tôpico Marcio
No site www.livfusp.com.br você encontra alguns livros da MIR. Já comprei  
com eles e é tranqüilo.


[]s,

Márcio.

On Sat, 03 Sep 2005 23:16:37 -0700, Jefferson Franca  
[EMAIL PROTECTED] wrote:


Aos amigos da lista que estão interessados em adquirir livros da extinta  
editora mir, um dos endereços é www.mir.ru  outro é
www.urss.ru . Certamente , um dos meus melhores livros de cálculo foi o  
demidovitch , eu o indico a todos que querem uma boa preparação nesse  
ramo da matemática.

Abraços



-
 Yahoo! Messenger com voz: PROMOÇÃO VOCÊ PODE LEVAR UMA VIAGEM NA  
CONVERSA. Participe!




--
Using Opera's revolutionary e-mail client: http://www.opera.com/mail/
=
Instruções para entrar na lista, sair da lista e usar a lista em
http://www.mat.puc-rio.br/~nicolau/olimp/obm-l.html
=


Re: [obm-l] Livros da MIR

2005-09-04 Por tôpico Marcio
De fato, passei o endereço errado. O correto (testei antes de escrever) é  
realmente www.livifusp.com.br.


[]s,

Márcio.

On Sun, 04 Sep 2005 14:06:12 -0700, Bernardo Freitas Paulo da Costa  
[EMAIL PROTECTED] wrote:



Cara, o site mudou de nome (tinha uma pagina velha que ainda tinha
isso, a unica no Google, e que disse que isso era a Livraria de Fisica
da Usp), agora tem um i a mais: http://www.livifusp.com.br/. Espero
que seja este mesmo.

Abraços,




--
Using Opera's revolutionary e-mail client: http://www.opera.com/mail/
=
Instruções para entrar na lista, sair da lista e usar a lista em
http://www.mat.puc-rio.br/~nicolau/olimp/obm-l.html
=


Re: [obm-l] Mais uma questão da prova.

2005-09-02 Por tôpico Marcio M Rocha

Rejane escreveu:


Quem puder me ajudar, eu agradeço.

 


Abraços.

 


Rejane

 


Questão 08)

 

No triângulo *ABC* ao lado, se *M* e *N* são pontos médios e a área do 
triangulo *DMC* é 1 dm², então a área, em dm², no triangulo *ABD* é:


 

A) 3  B) 2   C) 2,5  D) 
1,5  E) 1,9



  *M*

 



  *D*

 



  *N*

 



  *B*

 



  *C*

 



  *A*

 

 














Rejane, por falta de tempo devo ter escrito excessivamente, mas aí vai.

   Se a área de *DMC* é igual a 1, a área de DMB também é, pois os dois 
triângulos considerados têm mesma base e mesma altura. Daí, *Área *de 
*BDC* = 2. Como D é o baricentro de *ABC*, *BD*/*DN* = 2, e, por 
conseqüência, *Área* de *BDC* / *Área* de *DCN* = 2, ou seja, *Área* de 
*DCN* = 1. Isso significa que *Área* de *BCN* = 2 + 1 = 3. A Área de 
*ABN* = 3, pois N é médio de *AC*. A área de *ABD* = 2/3 da área de 
*ABN*, ou seja:


   *Área* de *ABD* = 2.

   Dê uma conferida, por favor.

[]s,

Márcio.
=
Instruções para entrar na lista, sair da lista e usar a lista em
http://www.mat.puc-rio.br/~nicolau/olimp/obm-l.html
=


[obm-l] Re: [obm-l] alguém pelo amor de deus consegue achar a soma dessa sequência????

2005-08-27 Por tôpico Marcio Cohen

para a linha n, dá combinação(2n,n)

- Original Message - 
From: Danilo Araújo Silva [EMAIL PROTECTED]

To: obm-l@mat.puc-rio.br
Sent: Saturday, August 27, 2005 2:44 PM
Subject: [obm-l] alguém pelo amor de deus consegue achar a soma dessa 
sequência




a sequência é simples...
é o trianculo de pascal com seus termos ao quadrado...
exemplo...
sabemos que...
1  =1
1 1=2
1 2 1 =4
1 3 3 1  =8
1 4 6 4 1   =16
1 5 10 10 5 1 =32
a soma das linhas do triangulo de pascal é de 2^n...
mas qual é soma de
1*1
1*11*1
1*1  2*2  1*1
1*1  3*3  3*3  1*1
1*1  4*4  6*6  4*4  1*1
para a n-ésima linha
alguém aí poderia me ajudar...
(eu não consigo condensar a a fórmula dada pelo  'meu' triangulo
evanescente...)
muito obrigado pela atenção...
se algu´´em achar a fórmula eu ficaria feliz se explicitasse o
procedimento usado para achar...
Tchau...Abração pros cês...



--
Lord Lestat vive...hum...

=
Instruções para entrar na lista, sair da lista e usar a lista em
http://www.mat.puc-rio.br/~nicolau/olimp/obm-l.html
=



=
Instruções para entrar na lista, sair da lista e usar a lista em
http://www.mat.puc-rio.br/~nicolau/olimp/obm-l.html
=


Re: [obm-l] Consulta sobre livros

2005-08-26 Por tôpico Marcio

Obrigado ao Carlos e ao Dirichilet pelas respostas.

[]s,

Márcio.


On Thu, 25 Aug 2005 16:57:36 -0700, Carlos Gomes [EMAIL PROTECTED]  
wrote:


Márcio, se o seu objetivo são problemas para teinamento básico de  
olimpiadas pode comprar eu os comprei e não me arrependi, são muito  
bons, excelentesproblemas e muito bem organizados


Cgomes
- Original Message - From: Marcio [EMAIL PROTECTED]
To: obm-l@mat.puc-rio.br
Sent: Thursday, August 25, 2005 11:22 PM
Subject: [obm-l] Consulta sobre livros


Boa tarde, pessoal.

Estive olhando a página www.artofproblemsolving.com e estou querendo
comprar os dois volumes do livro The Art of Problem Solving, de Sandor
Lehoczky e Richard Rusczyk.

Alguém os recomenda?

[]s,

Márcio.





--
Using Opera's revolutionary e-mail client: http://www.opera.com/mail/
=
Instruções para entrar na lista, sair da lista e usar a lista em
http://www.mat.puc-rio.br/~nicolau/olimp/obm-l.html
=


[obm-l] Consulta sobre livros

2005-08-25 Por tôpico Marcio

Boa tarde, pessoal.

Estive olhando a página www.artofproblemsolving.com e estou querendo  
comprar os dois volumes do livro The Art of Problem Solving, de Sandor  
Lehoczky e Richard Rusczyk.


Alguém os recomenda?

[]s,

Márcio.

--
Using Opera's revolutionary e-mail client: http://www.opera.com/mail/
=
Instruções para entrar na lista, sair da lista e usar a lista em
http://www.mat.puc-rio.br/~nicolau/olimp/obm-l.html
=


Re: [obm-l] geo plana

2005-08-23 Por tôpico Marcio
Um dos lados não paralelos mede 2r, e o outro, mede 2(r+1). Sejam x e y os  
dois lados paralelos, com x  y. Como o quadrilátero é circunscritível, a  
soma de lados opostos é constante.

Então, x + y = 2r + 2(r+1), ou seja, r = 4 cm.

Considere, agora, o triângulo retângulo cuja hipotenusa mede 2(r+1) e os  
catetos medem 2r e y - x. Fazendo as contas você chega a y - x = 6. Como x  
+ y = 18, então x = 6 e y = 12.

Os lados medem 6 cm, 8 cm, 10 cm e 12 cm, sendo que a = 6 cm.

A resposta é C.

[]s,

Márcio.



On Tue, 23 Aug 2005 10:07:53 -0700, Renato Bettiol  
[EMAIL PROTECTED] wrote:



(ITA-SP) Num trapezio retangulo circunscritivel, a soma dos dois lados
paralelos eh igual a 18cm, e a diferença dos dois outros lados eh
igual a 2cm. Se r eh o raio da circunferencia inscrita e a eh o
comprimento do menor lado do trapezio, entao a soma a+r (em cm) eh
igual a:
A)12
B)11
C)10
D)9
E)8

=
Instruções para entrar na lista, sair da lista e usar a lista em
http://www.mat.puc-rio.br/~nicolau/olimp/obm-l.html
=




--
Using Opera's revolutionary e-mail client: http://www.opera.com/mail/
=
Instruções para entrar na lista, sair da lista e usar a lista em
http://www.mat.puc-rio.br/~nicolau/olimp/obm-l.html
=


Re: [obm-l] Ola podem me ajudar??

2005-08-23 Por tôpico Marcio
	Apesar de considerar que aquilo que o Dirichilet escreveu é matemática  
(embora não tenha contas), vou fazer a parte braçal.


Cn,p-1 = (n!)/(p-1)!(n-p+1)!

Cn,p = (n!)/p!(n-p)!

O mmc dos denominadores é p!(n-p+1)!. Logo

	Cn,p-1 + Cn,p = [(n!)p + n!(n-p+1)]/p!(n-p+1)! = [n!(p+n-p+1)]/p!(n-p+1)!  
= [(n+1)!]/p!(n+1-p)! = Cn+1,p


	Só um comentário pessoal: se eu fosse corrigir uma questão dessas,  
ficaria muito feliz em encontrar uma solução que se baseasse apenas em  
argumento combinatório.


[]s,

Márcio.


On Tue, 23 Aug 2005 14:02:49 -0700, RAfitcho [EMAIL PROTECTED] wrote:

é muito pratica essa resposta.. mas nao posso escrever isso em uma prova  
discursiva se é que me entende...

preciso da parte matemática...
mas obrigado po tentar


- Original Message - From: Johann Peter Gustav Lejeune  
Dirichlet [EMAIL PROTECTED]

To: obm-l@mat.puc-rio.br
Sent: Tuesday, August 23, 2005 1:59 AM
Subject: Re: [obm-l] Ola podem me ajudar??


Ha um argumento combinatorio que eu adoro!

Temos que provar que

o tanto de jeitos de se escolher p bolas de bilhar de
um conjunto de n+1

e

o tanto de jeitos de se escolher p bolas de bilhar de
um conjunto de n,

mais

o tanto de jeitos de se escolher p-1 bolas de bilhar
de um conjunto de n.

Bem, podemos fazer assim:
*Marcar uma bola.

Ai temos udas possibilidades:

--Jogar esta bola fora, o que da a segunda parte
acima;

--Escolher esta bola como uma das p que eu quero, e
escolher as outras depois, o que da a segunda frase.

E ai, convenceu?




--- RAfitcho [EMAIL PROTECTED] escreveu:


   a) Mostre que Cn,p-1 + Cn,p = Cn+1,p


nao to consguindo...
obrigado desde já







___
Yahoo! Acesso Grátis - Internet rápida e grátis.
Instale o discador agora! http://br.acesso.yahoo.com/
=
Instruções para entrar na lista, sair da lista e usar a lista em
http://www.mat.puc-rio.br/~nicolau/olimp/obm-l.html
=


=
Instruções para entrar na lista, sair da lista e usar a lista em
http://www.mat.puc-rio.br/~nicolau/olimp/obm-l.html
=




--
Using Opera's revolutionary e-mail client: http://www.opera.com/mail/
=
Instruções para entrar na lista, sair da lista e usar a lista em
http://www.mat.puc-rio.br/~nicolau/olimp/obm-l.html
=


Re: [obm-l] ESA - AJUDA

2005-08-19 Por tôpico Marcio

Oi Sharon,

Coloque a origem do sistema de coordenadas no local onde se localiza o  
menino. Desse modo, o topo do muro (e que, pelo problema, pertence à  
parábola) se localiza no ponto (6, 3).


Faça a substituição na lei da função y = ax^2 + (1- 4a)x, e você obterá a  
= -1/4. Daí, é só calcular o valor máximo da função.


Eu encontrei 4.

Qualquer coisa, estamos aí.

[]s,

Márcio.




On Thu, 18 Aug 2005 23:37:05 -0700, Sharon Guedes [EMAIL PROTECTED]  
wrote:




Olá pessoal,


Um amigo meu fez a prova da ESA e teve dúvidas em resolver essa questão.  
Resolvi ela mas acabei chegando em duas respostas.


Alguém poderia nos ajudar?


Questão:




Estando afastado 6 metros de um muro de 3 metros de altura, um menino  
chuta a bola que cai exatamente sobre o citado muro, após percorrer a  
trajetória descrita pela equação y= ax2 + (1-4a)x, em relação ao sistema  
de coordenadas usual. Nestas condições, a altura máxima atingida pela  
bola é:



(a) 8(b) 6  (c) 4   (d) 12  (e) 10
Atenciosamenta,
Sharon Guedes.

__
Converse com seus amigos em tempo real com o Yahoo! Messenger
http://br.download.yahoo.com/messenger/




--
Using Opera's revolutionary e-mail client: http://www.opera.com/mail/
=
Instruções para entrar na lista, sair da lista e usar a lista em
http://www.mat.puc-rio.br/~nicolau/olimp/obm-l.html
=


Re: [obm-l] trigonometria

2005-08-17 Por tôpico Marcio

Boa tarde a todos.

Caro Jefferson,

Vamos, em primeiro lugar, lembrar a questão original, que era mais ou  
menos assim: As tangentes dos três ângulos internos de um triângulo são  
números inteiros e positivos. Calcule seus valores.


Eu propus uma solução chegando ao terno (1, 2, 3), mas havia afirmado que  
não sabia se a solução era única. Depois, um colega provou que não é  
possível que duas das tangentes sejam maiores que 2, mostrando, então, que  
a solução (1, 2, 3) é única.


Se o problema que você está resolvendo é esse, as suas duas conjecturas  
são falsas.


Suponhamos que bc = 3, ac = 3 e ab = 3. Das duas primeiras igualdades vem  
que a/b = 1, ou seja, a = b, que usado na terceira igualdade produz a = b  
= srqt(3), valores não inteiros.


Uma situação análoga acontece na sua segunda suposição.

Sendo assim, parece-me que o problema tem apenas uma solução.

[]s,

Márcio.



On Tue, 16 Aug 2005 19:13:02 -0700, Jefferson Franca  
[EMAIL PROTECTED] wrote:



Meu caro amigo e ex aluno Eurico, vc não deve ter pensado no seguinte:
a + b + c = a.b.c = (1/bc) + (1/ac) + (1/ab) = 1  e, suponha que bc =  
3, ac = 3 e ab = 3, e se tivéssemos bc = 2, ac = 4 e ab = 4?
Eu, sinceramente acho que esta questão não tem só uma solução como vc  
afirma.

Valeu e abraço

Antonio Eurico Dias [EMAIL PROTECTED] escreveu:

E completando o raciocinio do Dirichlet:

TgA + TgB + TgC = TgA. TgB. TgC

a + b + c = a.b.c  =  abc - a = b + c = a (bc - 1) = b + c

a = (b + c) / (bc - 1), como a é inteiro positivo: b + c = bc - 1  =   
bc - b = c + 1


b.(c - 1) = c + 1   =  b = (c+1)/(c-1)   =   b = 1 +   2/(c-1)

Logo, como b também é inteiro   c - 1 = 2 ou c -1 = 1, que levam ao  
unico conjunto de solucoes inteiras positivas


(1 , 2 , 3)...


Eurico

Sistema Elite de Ensino - Unidade Belém



-
Start your day with Yahoo! - make it your home page
__
Converse com seus amigos em tempo real com o Yahoo! Messenger
http://br.download.yahoo.com/messenger/




--
Using Opera's revolutionary e-mail client: http://www.opera.com/mail/
=
Instruções para entrar na lista, sair da lista e usar a lista em
http://www.mat.puc-rio.br/~nicolau/olimp/obm-l.html
=


Re: [obm-l] ajuda questão de Física ITA

2005-08-11 Por tôpico marcio aparecido
não consegui responder graficamente, alguem pode da uma ajuda!!

=
Instruções para entrar na lista, sair da lista e usar a lista em
http://www.mat.puc-rio.br/~nicolau/olimp/obm-l.html
=


Re: [obm-l] trigonometria

2005-08-10 Por tôpico Marcio
On Wed, 10 Aug 2005 06:20:13 -0700, Jefferson Franca  
[EMAIL PROTECTED] wrote:


Será que alguém já viu esta questão ou tem alguma idéia de como resolver  
?
Sejam a ,b e c ângulos internos de umtriângulo e, supondo que as  
tangentes dos três ângulos sejam números inteiros e positivos, calcule  
essas tangentes.

Valeu

__
Converse com seus amigos em tempo real com o Yahoo! Messenger
http://br.download.yahoo.com/messenger/




--
Using Opera's revolutionary e-mail client: http://www.opera.com/mail/

Oi, Jefferson.

Se não errei nada, aqui vai.

Ângulos: a, b e c

a + b + c = 180 = tg(a + b + c)= tg 180, ou seja, tg(a + b + c) = 0

Daí, tg(a + b) + tg(c) = 0.

No final das contas, chega-se a

tg a + tg b + tg c = (tg a)(tg b)(tg c)

Como as tangentes são números inteiros e positivos, uma opção (não sei se  
única) é


tg a = 1, tg b = 2 e tg c = 3


[]s,

Márcio.
=
Instruções para entrar na lista, sair da lista e usar a lista em
http://www.mat.puc-rio.br/~nicolau/olimp/obm-l.html
=


Re: [obm-l] Economia Matemática

2005-08-06 Por tôpico Marcio M Rocha

Maurício escreveu:


 Oi, sou eu de novo.

 Estou interessado em fazer uma pós na área de
Economia Matemática. Vocês sabem onde se faz pesquisa
de qualidade nessa área aqui no Brasil ou no Exterior?

 Abraços e obrigado,
 Maurício


__
Do You Yahoo!?
Tired of spam?  Yahoo! Mail has the best spam protection around 
http://mail.yahoo.com 
=

Instruções para entrar na lista, sair da lista e usar a lista em
http://www.mat.puc-rio.br/~nicolau/olimp/obm-l.html
=

 

Se você procura qualidade, penso que não se pode deixar de mencionar o 
IMPA, que tem cursos de mestrado e doutorado em Economia Matemática. 
Veja em www.impa.br.


Márcio.
=
Instruções para entrar na lista, sair da lista e usar a lista em
http://www.mat.puc-rio.br/~nicolau/olimp/obm-l.html
=


Re: [obm-l] videos impa

2005-07-30 Por tôpico marcio aparecido
eu não consegui baixar nenhum!!!

=
Instruções para entrar na lista, sair da lista e usar a lista em
http://www.mat.puc-rio.br/~nicolau/olimp/obm-l.html
=


Re: [obm-l] Prova da IMO - Primeiro dia - Solucoes

2005-07-28 Por tôpico Marcio Cohen
  Oi gente! Esse ano não pude pensar nos problemas da imo do jeito que 
gosto (pegando a prova logo depois de ela ser liberada no mathlinks e indo 
para um restaurante pensar 4h30m direto nela :)).. Mas finalmente peguei a 
prova (do primeiro dia) de jeito e consegui fazer as questoes. Vou mandar 
aqui minhas solucoes do 1o dia conforme o gugu sugeriu (obs: reparei que 
minha solucao da 2 eh mto parecida com a do gugu, mas agora vou mandar mesmo 
assim). Espero que esteja tudo certo (minha unica duvida eh na 2 :), mas 
acho que tá certa. prefiro mandar logo).

   Vou colocar as soluções após os enunciados do shine também.
Obs: Quem não entender a minha solução da 3 deve aguardar um artigo na 
eureka sobre desigualdades e contas simétricas que estou escrevendo!


- Original Message - 
From: Carlos Gustavo Tamm de Araujo Moreira [EMAIL PROTECTED]

To: obm-l@mat.puc-rio.br
Sent: Sunday, July 24, 2005 12:06 AM
Subject: Re: [obm-l] Prova da IMO - Primeiro dia - Solucoes



  Oi pessoal,
  Resolvi compilar as minhas soluções de cada um dos dias para fins de
referência (em particular porque algumas de minhas mensagens anteriores
foram um pouco confusas, ou por não ter a solução junto ou por não dizerem
no subject sobre que problema tratavam). Seguem aqui (como sempre, após a
mensagem original do Shine) as soluções do primeiro dia.
  Abraços,
   Gugu


Oi gente,

Acabei de ver a primeira prova da IMO no site
http://www.mathlinks.ro/

Lá vão os enunciados (eu mesmo traduzi agora).

1. Escolhemos seis pontos sobre os lados do triângulo
equilátero ABC: A_1, A_2 sobre BC; B_1, B_2 sobre AC;
C_1, C_2 sobre AB. Essa escolha é feita de modo que
A_1A_2B_1B_2C_1C_2 é um hexágono convexo com todos os
seus lados iguais.

Prove que A_1B_2, B_1C_2 e C_1A_2 são concorrentes.


Solução: Spg, ponha A1 = (x,0); A2 = (-y,0), onde x,y sao reais positivos 
com x+y = r, r o lado do hexagono.
  Sejam a e b os angulos B1,A2,C e C2,A1,B respectivamente (i.e, os 
angulos do hexagono com a base BC).
  Pensando vetorialmente, temos C1 = (x+rcos(b)-rcos60, rsen(b)+rsen60); B2 
= (-y-rcos(a)+rcos60, rsen(a)+rsen60).
  Como todos os lados sao iguais, a distancia de C1 a B2 eh r. Usando que 
x+y=r, cos60 = 1/2 e a formula da distancia entre dois pontos isso dá: 
(cos(b)+cos(a))^2 + (sen(b)-sen(a))^2 = 1 donde cos(a+b) = -1/2 e a+b = 120 
graus.
   Esse mesmo raciocínio se aplica a cada uma das bases, e portanto os 
triângulos das pontas são todos congruentes.
   Todos os lados do triangulo B2,C2,A2 são iguais (cada um deles eh base 
de um triangulo de lados r,r com angulo 120-b entre esses r´s) e portanto 
ele eh equilatero. As retas B1C2, A1B2, C1A2 são alturas desse triangulos 
equilatero (a congruencia LLL dos triangulos B1B2C2 e B1A2C2 implicam C2B1 
perpendicular a B2A2), e portanto concorrem num ponto.



2. Seja a_1,a_2,... uma seqüência de inteiros com

infinitos termos positivos e negativos. Suponha que
para todo n inteiro positivo os números
a_1,a_2,...,a_n deixam n restos diferentes na divisão
por n.

Prove que todo inteiro aparece exatamente uma vez na
seqüência a_1,a_2,...


Solução:
  Basta provar que o 0 aparece na sequencia, pois r aparece na sequencia 
(x_n) sse 0 aparece na sequencia (x_n - r), e (x_n) tem as propriedades do 
enunciado sse (x_n - r) tem.

  Suponha, spg, x1 = p  0 (se x10, olhe para a sequencia (- x_n) ):
 Temos entao x2 = p-1 ou x2 = p+1 (pq se x2 = p+i com i impar 1, entao 
x2=x1 (mod i) e i =3 ainda nao chegou!).
 Vamos provar por inducao que a sequencia nunca da saltos, i.e, que 
cada novo termo está a uma unidade de distancia de algum termo que ja 
apareceu na sequencia. Como a sequencia eventualmente fica negativa, isso 
garante que ela passa pelo zero.
 Suponha que {x1,x2,...,xt} = {p-s,...,p-2,p-1,p,p+1,p+2,...,p+r} com 
r+s+1=t.
 Entao, as unicas opcoes para x(t+1) sao p-s-1 e p+r+1  (observe que 
p+r+1=p-s-1 (mod t+1)).
 De fato, olhando mod (t+1) temos que {p-s-1,p-s,...,p+r} e {p-s, 
...,p+r,p+r+1} formam um sistema completo de residuos mod(t+1) (pois sao uma 
versao transladada de {1,2,...,r+s+2=t+1}). Os numeros que poderiam ocupar a 
posicao x(t+1) sao portanto os que tem a mesma classe que estes dois tem mod 
(t+1), i.e, sao: p+r+1, p+r+1+(t+1), p+r+1+2(t+1), ..., p-s-1, p-s-1-(t+1), 
p-s-1-2(t+1), ...
 Agora, se x(t+1) = p+r+1+k(t+1) com k1, entao x(t+1)=p+r (mod 
1+k(t+1)), e como 1+k(t+1)  t+1, isso vai estragar a sequencia na posicao 
1+k(t+1). O outro caso eh analogo.



3. Sejam x,y,z reais positivos tais que xyz = 1.
Prove que
(x^5-x^2)/(x^5+y^2+z^2) +
(y^5-y^2)/(x^2+y^5+z^2) +
(z^5-z^2)/(x^2+y^2+z^5) = 0.


Solução:
  Tirando mmc e olhando para as somas simétricas, a desigualdade eh 
equivalente a:

   sym_sum (x^5-x^2)(x^2+y^5+z^2)(x^2+y^2+z^5) = 0
   Como (x^2+y^5+z^2)(x^2+y^2+z^5) = x^4 +(y^5+z^2+y^2+z^5)x^2 + 
(y^5+z^2)(z^5+y^2), o que quero eh:
   sym_sum (x^9 + 2x^7y^5 + 2x^7y^2 + 2x^5y^7 + x^5y^5z^5 + x^5y^2z^2) 
=

   sym_sum 

[obm-l] Resultado IMC

2005-07-27 Por tôpico Marcio Cohen



 Saiu o resultado oficial da IMC de 
2005 (a competição internacional de matemática universitária)! O Brasil foi 
incrivelmente bem, o melhor resultado da história!!!
 O Alex (ufrj) foi grand first prize! 
Esse é um prêmio especial dado aos melhores dentre os primeiros colocados. O 
Brasil (e se não me engano a America Latina) nunca tinha ganho.
 Em seguida, Fabinho (puc)e 
Thiago Barros (unicamp) ficaram com First Prize!
 Bernardo (ufrj/politechinique), Thiago 
Sobral (ita) e Carlos Stein (ita) ganharam Second Prize! 
 Além desses, o Brasil teve third prize 
e menção honrosa, mas não sei exatamente como ficou isso após a 
reclassificação!
 Abraços a todos,
 Marcio


Re: [obm-l] ajuda questão de Física ITA

2005-07-26 Por tôpico marcio aparecido
tem como você fazer um esboço do gráfico ai Roberto ??

=
Instruções para entrar na lista, sair da lista e usar a lista em
http://www.mat.puc-rio.br/~nicolau/olimp/obm-l.html
=


Re: [obm-l] geometria

2005-07-25 Por tôpico Marcio M Rocha

Desculpe-me, Saulo, mas os ângulos são congruentes sim. Veja:

QBP é um ângulo formado pela corda BP e pelo lado AB, que é tangente à 
circunferência. Logo, mede metade do arco menor BP. O ângulo PCB é um 
ângulo inscrito e também mede metade do arco menor BP. Uma argumentação 
parecida vale para os ângulos PCR e PBD.


Outra coisa: quem disse que BC é um diâmetro?

Dê uma conferida com cuidado, por favor.

[]s,

Márcio.



saulo nilson escreveu:


os angulos nao sao congruentes BC nao passa pelo centro da
circunferencia, BC e uma corda e nao um diametro.
Eu fiz achando o raio da circunferencia que e 13, dai vc acha os lados
e pela area do triangulo isosceles vc acha acha o valor da altura
pedida, mas na da uma resposta simples.

Eu projetei PQ e PR sobre os raios que unem o centro aos pontos de
tangencia da circunferencia ai obtive um quadrilatero que tem lados
r-9 e r-4 que e semelhante a AQPR, dai vc acha o raio que da 13, isso
esta certo ou errado?
On 7/20/05, Marcio M Rocha [EMAIL PROTECTED] wrote:
 


Eder Albuquerque escreveu:

   


Olá,

Gostaria de ajuda no seguinte problema: seja ABC um triângulo
isósceles, onde AB=AC são tangentes a uma circunferência e BC é uma
corda. Seja P um ponto sobre a circunferência anterior, interno ao
triângulo  ABC, tal que a distância de P a AB é 9 e a distância de P a
AC é 4. Encontre a distância de P a BC.

Não tô conseguindo resolver...

Grato,

Eder

__
Converse com seus amigos em tempo real com o Yahoo! Messenger
http://br.download.yahoo.com/messenger/

 


A resposta é 6.

Sejam Q, R e D os pés das perpendiculares, respectivamente a AB, AC e BC
por P.  Construa o triângulo BPC. Veja que os ângulos QBP e PCB são
conguentes. Daí, os triângulos retângulos QPB e  PDC  são semelhantes, e
podemos escrever que (PQ / PD) = (PB / PC).

Analogamente, os ângulos PCR e PBC são congruentes, donde vem a
semelhança dos triângulos retângulos PCR e PBD, e podemos escrever que
(PR / PD) = (PC / PB).

Logo, (PQ / PD) = (PD / PR), ou seja,  (PD)^2 = (PQ).(PR).

Pelo problema, PQ = 9  e  PR = 4. Assim, PD = 6.

Este problema consta do livro Challenging Problems in Geometry.

Um abraço,

Márcio.
=
Instruções para entrar na lista, sair da lista e usar a lista em
http://www.mat.puc-rio.br/~nicolau/olimp/obm-l.html
=

   



=
Instruções para entrar na lista, sair da lista e usar a lista em
http://www.mat.puc-rio.br/~nicolau/olimp/obm-l.html
=

 



=
Instruções para entrar na lista, sair da lista e usar a lista em
http://www.mat.puc-rio.br/~nicolau/olimp/obm-l.html
=


[obm-l] ajuda questão de Física ITA

2005-07-25 Por tôpico marcio aparecido
(ITA) Três turista, reunidos num mesmo local e dispondo de uma
bicicleta que pode levar somente duas pessoa de cada vez, precisando
chegar a um centro turístico o mais rápido possível. O turista A leva
turista B, de bicicleta, até um ponto X do percurso e retorna para
apanhar o turista C que vinha caminhando ao seu encontro. O turista B,
a partir de X continua a pé sua viagem rumo ao centro turístico.
Os três chegam simultaneamente juntos ao centro turítico.
A velocidade média como pedestre é V1, enquanto como ciclista e V2.
Com que velocidade média os turistas farão o percurso total ??

=
Instruções para entrar na lista, sair da lista e usar a lista em
http://www.mat.puc-rio.br/~nicolau/olimp/obm-l.html
=


Re: [obm-l] geometria

2005-07-19 Por tôpico Marcio M Rocha

Eder Albuquerque escreveu:


Olá,
 
Gostaria de ajuda no seguinte problema: seja ABC um triângulo 
isósceles, onde AB=AC são tangentes a uma circunferência e BC é uma 
corda. Seja P um ponto sobre a circunferência anterior, interno ao 
triângulo  ABC, tal que a distância de P a AB é 9 e a distância de P a 
AC é 4. Encontre a distância de P a BC.
 
Não tô conseguindo resolver...
 
Grato,
 
Eder


__
Converse com seus amigos em tempo real com o Yahoo! Messenger
http://br.download.yahoo.com/messenger/


A resposta é 6.

Sejam Q, R e D os pés das perpendiculares, respectivamente a AB, AC e BC 
por P.  Construa o triângulo BPC. Veja que os ângulos QBP e PCB são 
conguentes. Daí, os triângulos retângulos QPB e  PDC  são semelhantes, e 
podemos escrever que (PQ / PD) = (PB / PC).


Analogamente, os ângulos PCR e PBC são congruentes, donde vem a 
semelhança dos triângulos retângulos PCR e PBD, e podemos escrever que 
(PR / PD) = (PC / PB).


Logo, (PQ / PD) = (PD / PR), ou seja,  (PD)^2 = (PQ).(PR).

Pelo problema, PQ = 9  e  PR = 4. Assim, PD = 6.

Este problema consta do livro Challenging Problems in Geometry.

Um abraço,

Márcio.
=
Instruções para entrar na lista, sair da lista e usar a lista em
http://www.mat.puc-rio.br/~nicolau/olimp/obm-l.html
=


Re: [obm-l] geometria plana

2005-07-19 Por tôpico Marcio M Rocha

elton francisco ferreira escreveu:


Olá pessoal, ai vai algumas questoes que começei a a
resoluçao mas nao consigo terminar. Se vcs puderem me
ajudarem.

1 - Num triangulo retangulo ABC, sabe-se que a área
vale 2s e que a razão entre os catetos é b/c=k.
Calcule seus lados.

2 – A diferença entre os catetos de um triangulo
retângulo é d e a área é s. Determine os catetos para:
d = 17 e s = 84
3 – Num triangulo retângulo, a hipotenusa é a e a
altura relativa a ela é h. Calcule os catetos para : a
= 25 e h = 12






___ 
Yahoo! Acesso Grátis - Internet rápida e grátis. 
Instale o discador agora! http://br.acesso.yahoo.com/

=
Instruções para entrar na lista, sair da lista e usar a lista em
http://www.mat.puc-rio.br/~nicolau/olimp/obm-l.html
=

 

1- A área é a metade de bc, ou seja, bc = 4s. Multiplicando ambos os 
membros de b/c = k por bc, vem


b^2 = bck, ou seja, b = sqrt(4ks), ou b = 2.sqrt(ks). Daí, c = 
2.sqrt(ks) / k


2- b = 24 e c = 7. Como o produto dos catetos é o dobro da área, temos 
que bc = 168. Como b - c = 17, você cai numa eq do 2o grau.


3- b = 20 e c = 15. Veja que b^2 + c^2 = 625 e bc = 300. Daí, b + c = 
35, e você monta uma equação do 2o grau.


Abraços,

Márcio.
=
Instruções para entrar na lista, sair da lista e usar a lista em
http://www.mat.puc-rio.br/~nicolau/olimp/obm-l.html
=


Re: [obm-l] Re: [obm-l] OFF TOPIC: programa de caraceteres matemáticos

2005-07-17 Por tôpico marcio aparecido
onde eu consigo esse LaTeX

=
Instruções para entrar na lista, sair da lista e usar a lista em
http://www.mat.puc-rio.br/~nicolau/olimp/obm-l.html
=


Re: [obm-l] Matrizes

2005-07-14 Por tôpico marcio aparecido
não entendi!!

=
Instruções para entrar na lista, sair da lista e usar a lista em
http://www.mat.puc-rio.br/~nicolau/olimp/obm-l.html
=


[obm-l] Matrizes

2005-07-13 Por tôpico marcio aparecido
ajuda com a seguinte questão, ai vai o link dela:

http://mas-usp.sites.uol.com.br/matriz.JPG

=
Instruções para entrar na lista, sair da lista e usar a lista em
http://www.mat.puc-rio.br/~nicolau/olimp/obm-l.html
=


[obm-l] Indicação Livro

2005-07-11 Por tôpico marcio aparecido
alguem pode me indicar algum livro a nível de ensino médio para física
e matemática, que tenha um bom conteudo, que não seja muito resumido.
Valew!!

=
Instruções para entrar na lista, sair da lista e usar a lista em
http://www.mat.puc-rio.br/~nicolau/olimp/obm-l.html
=


Re: [obm-l] Uma desigualdade legal!

2005-07-10 Por tôpico Marcio Cohen
É realmente complicado calcular essa derivada.. Uma possível solução para 
esse problema é simplesmente tirar o mmc.. Aqui está:


Vc quer provar que
  sym_sum (a^(x+2) + 1) / (a^x bc + 1) = 6
E as passagens abaixo são equivalentes:

sym_sum (a^(x+2) + 1)(b^x ac + 1)(c^x ab + 1) = 6(a^x bc + 1)(b^x ac + 
1)(c^x ab + 1)


sym_sum  (a^(x+4) b^(x+1) c^(x+1) + 2 * a^(x+3) b^x c + a^(x+2) + b^(x+1) 
c^(x+1) a^2 + 2*a^x bc + 1 )
   = sym_sum ( a^(x+2) * b^(x+2) * c^(x+2) +  3*a^(x+1) b^(x+1)c^2 + 3*a^x 
bc + 1)


Agora, pela desigualdade de muirhead (bunching), voce sabe que:

sym_sum [a^(x+4) b^(x+1) c^(x+1) - a^(x+2) * b^(x+2) * c^(x+2)] = 0
sym_sum [2 * a^(x+3) b^x c - 2*a^(x+1) b^(x+1)c^2]  =0
sym_sum [b^(x+1) c^(x+1) a^2 - a^(x+1) b^(x+1)c^2] = 0
sym_sum [a^(x+2) - a^x bc ] = 0

Somando tudo voce conclui a desigualdade pedida.

Abraços,
Marcio

- Original Message - 
From: Marcos Martinelli [EMAIL PROTECTED]

To: obm-l@mat.puc-rio.br
Sent: Sunday, July 10, 2005 4:22 PM
Subject: [obm-l] Uma desigualdade legal!



Boa tarde pessoal. Precisco de ajuda nessa desigualdade. Lá vai:

Dados a,b,c,x reais positivos provar que:

[a^(x+2)+1]/[a^(x)*b*c+1]+[b^(x+2)+1]/[b^(x)*a*c+1]+[c^(x+2)+1]/[c^(x)*b*a+1]=3.

Tentei resolver através da desigualdade de Jensen, considerando a
seguinte função
f(u)=[u^(x+2)+1]/[k*u^(x-1)+1], onde k=a*b*c. Assumindo que a segunda
derivada dessa função é positiva a desigualdade acima é imediata. Meu
problema foi demonstrar que essa segunda derivada é sempre positiva
para qualquer u positivo e x positivo. Tentei derivar implicitamente
mas as contas crescem muito. Gostaria da ajuda de vocês e, quem sabe,
até uma outra solução pro problema. Obrigado!

=
Instruções para entrar na lista, sair da lista e usar a lista em
http://www.mat.puc-rio.br/~nicolau/olimp/obm-l.html
=





=
Instruções para entrar na lista, sair da lista e usar a lista em
http://www.mat.puc-rio.br/~nicolau/olimp/obm-l.html
=


Re: [obm-l] Problema

2005-06-28 Por tôpico Marcio M Rocha

Lincoln escreveu:


Alguém pode me dar uma ajuda neste problema?
 

Seja /ABCD/ um retângulo de lados /AB/ = 4 e /BC/ =3.  A perpendicular 
à diagonal /BD/ traçada por /A/ corta /BD/ no ponto /H/. Chamamos de 
/M/ o ponto médio de /BH/ e de /N/ o ponto médio de /CD/. Calcule a 
medida do segmento /MN/.


 

 


Lincoln,
Escrevi rápido e sem muita organização. Veja se você entende e se está 
tudo OK.


BD = 5 e AH = 2,4. A perpendicular ao segmento AH, passando por M, 
intersecta AB no ponto P. Desse modo, MP = 1,2 e P é o médio de AB. Os 
segmentos PN e BD cortam-se ao meio, num ponto Q.  Seja R um ponto de 
BD, simétrico de M em relação a Q. Os triângulos PMQ e NRQ são 
congruentes. Desse modo, MN^2 = RN^2 + RM^2, ou seja, MN^2 = (1,2)^2 + 
(1,8)^2. Daí, MN = 3*sqrt(13)/5.


[]s,

Márcio.
=
Instruções para entrar na lista, sair da lista e usar a lista em
http://www.mat.puc-rio.br/~nicolau/olimp/obm-l.html
=


[obm-l] Re: [obm-l] Re: [obm-l] Gabaritos do nivel universtirário

2005-06-19 Por tôpico Marcio Cohen
Basta entrar no site da obm e baixar as Eurekas. Nelas voc vai encontrar as 
solues do nvel universitrio das provas at 2003.


- Original Message - 
From: Daniel Regufe [EMAIL PROTECTED]

To: obm-l@mat.puc-rio.br
Sent: Sunday, June 19, 2005 10:31 PM
Subject: [obm-l] Re: [obm-l] Gabaritos do nivel universtirrio


infelizmente no site da OBM encontra-se apenas gabaritos da primeira fase 
da universitria ... eu queria os gabaritos da segunda fase ...


mas de qq forma
Muito obrigado


From: Jnior [EMAIL PROTECTED]
Reply-To: obm-l@mat.puc-rio.br
To: obm-l@mat.puc-rio.br
Subject: Re: [obm-l] Gabaritos do nivel universtirrio
Date: Sun, 19 Jun 2005 17:54:42 -0300

Todos os gabaritos de provas passadas estao em:
http://www.obm.org.br/frameset-provas.htm

Em 19/06/05, Daniel Regufe[EMAIL PROTECTED] escreveu:
 Aonde q eu posso encontrar os gabaritos das provas da obm do nivel
 universitrio da segunda fase ??

 Muito obrigado

 _
 MSN Messenger: converse online com seus amigos .
 http://messenger.msn.com.br


=
 Instrues para entrar na lista, sair da lista e usar a lista em
 http://www.mat.puc-rio.br/~nicolau/olimp/obm-l.html

=


=
Instrues para entrar na lista, sair da lista e usar a lista em
http://www.mat.puc-rio.br/~nicolau/olimp/obm-l.html
=


_
MSN Messenger: converse online com seus amigos . 
http://messenger.msn.com.br


=
Instrues para entrar na lista, sair da lista e usar a lista em
http://www.mat.puc-rio.br/~nicolau/olimp/obm-l.html
=





=
Instrues para entrar na lista, sair da lista e usar a lista em
http://www.mat.puc-rio.br/~nicolau/olimp/obm-l.html
=


Re: [obm-l] Dúvida elementar

2005-06-09 Por tôpico Marcio M Rocha

Luiz Ernesto Leitao escreveu:

Lendo o livro de Análise do Djairo Guedes ele pediu que se provasse a 
seguinte afirmativa elementar:

Prove que  p ( p natural) é par, se e somente se, p^2 for par.

__
Converse com seus amigos em tempo real com o Yahoo! Messenger
http://br.download.yahoo.com/messenger/


1a parte: p é par = p^2 é par

p é par = p = 2n

Donde se conclui que p^2 = 4(n^2) = 2(2n^2) que é par

2a parte: p^2 é par = p é par

Prove a contrapositiva: p é ímpar = p^2 é ímpar

p é ímpar = p = 2n +1

Daí,

p^2 = (2n + 1)^2 = 4n^2 + 4n + 1 = 2(2n^2 + 2n) + 1 que é ímpar.

Márcio.
=
Instruções para entrar na lista, sair da lista e usar a lista em
http://www.mat.puc-rio.br/~nicolau/olimp/obm-l.html
=


[obm-l] Re: [obm-l] teorema chinês do resto

2005-05-30 Por tôpico Marcio Cohen



Da primeira, x = 3 + 17k.
Na segunda, 3+17k = 10 (mod 16) = k = 7 (mod 
16) =  k = 7 + 16t = x = 3 + 17(7 + 16t) = 122 + 17*16t
Na terceira, 122 + 17*16t = 0 (mod 15) = 2 + 
2*1*t = 0 (mod15) = t = -1 (mod 15) = t = -1 + 15s
 = x = 122 + 17*16*(-1 + 15s) = x = 
-150 + 17*16*15s, ou x = 3930 (mod 4080)
 (todas as variáveis acima são 
inteiras)


  - Original Message - 
  From: 
  Guilherme Neves 
  To: obm-l@mat.puc-rio.br 
  Sent: Monday, May 30, 2005 1:45 PM
  Subject: [obm-l] teorema chinês do 
  resto
  
  
  alguem poderia resolver esse sistema? 
  
  x=3 (mod 17)
  x=10 (mod 16)
  x=0 (mod 15)
  
  * = (usei como´o símbolo de congruência)
  
  Chegou o que faltava: MSN Acesso Grátis Instale Já! 
  = 
  Instruções para entrar na lista, sair da lista e usar a lista em http://www.mat.puc-rio.br/~nicolau/olimp/obm-l.html 
  = 



[obm-l] ajuda com proporções

2005-05-27 Por tôpico marcio aparecido
ajuda com proporções:
quais são os possiveis valores da seguinte proporção:
a/b+c+d = b/a+c+d = c/a+b+d = d/a+b+c

=
Instruções para entrar na lista, sair da lista e usar a lista em
http://www.mat.puc-rio.br/~nicolau/olimp/obm-l.html
=


Re: [obm-l] Problemas russos

2005-05-24 Por tôpico Marcio M Rocha

fabiodjalma escreveu:

Onde você os encontrou? 



Em (09:21:52), obm-l@mat.puc-rio.br escreveu: 



 

Bom dia a todos! 

Encontrei 100 problemas russos traduzidos pelo Paulo Santa Rita e estou 
tentando resolvê-los. Gostaria de uma idéia para o seguinte: 

É dado um retangulo ABCD com o comprimento da diagonal AC valendo L. 
Quatro círculos com centros em A, B, C e D e raios respectivamente 
iguais a a, b, c e d, sao tais que : L  a + c , a + c = b + d. 
Prove que se pode inscrever um circulo no quadrilatero formado pelas 
interseccões entre duas tangentes comuns externas ao circulos A e C e 
duas tangentes comuns externas aos circulos B e D. 

Um grande abraço. 

[]s, 

Márcio. 
= 
Instruções para entrar na lista, sair da lista e usar a lista em 
http://www.mat.puc-rio.br/~nicolau/olimp/obm-l.html 
= 

-- 



 


Oi, Fábio,

Os problemas estão em http://www.mat.puc-rio.br/~nicolau/psr/ 
http://www.mat.puc-rio.br/%7Enicolau/psr/


Um abraço.

Márcio.
=
Instruções para entrar na lista, sair da lista e usar a lista em
http://www.mat.puc-rio.br/~nicolau/olimp/obm-l.html
=


Re: [obm-l] Re: [obm-l] Função

2005-05-03 Por tôpico Marcio M Rocha
Viviane Silva escreveu:

Como se resolve uma função do tipo. Este não é o exercício mas é 
parecido com este
 
1) f(3x+1)=x^2+3x+25
g(x+1)=2x+1
Encontre f(g(-1))
 
Grata


MSN Busca: fácil, rápido, direto ao ponto. Encontre o que você quiser. 
Clique aqui. http://g.msn.com/8HMABR/2740??PS=47575 
= 
Instruções para entrar na lista, sair da lista e usar a lista em 
http://www.mat.puc-rio.br/~nicolau/olimp/obm-l.html 
= 
g(-1) = g(-2 + 1) = 2*(-2) + 1 = -4 + 1 = -3
f(-3) = f(3*(-4/3) + 1) = (-4/3)^2 + 3*(-4/3) + 25 = 16/9 - 4 + 25 = (16 
+ 189)/9 = 205/9

Márcio.
=
Instruções para entrar na lista, sair da lista e usar a lista em
http://www.mat.puc-rio.br/~nicolau/olimp/obm-l.html
=


Re: [obm-l] Duvida

2005-04-21 Por tôpico Marcio Cohen
Oi Luiz!
Você trocou o sinal das desigualdades, essa solução está errada..
Segue uma solucao absurdamente feia (mas aparentemente correta) para o 
problema (desafio qualquer um a achar uma solução mais feia :))

Problema: a=1^2,  a+b=1^2+2^2,  a+b+c=1^2+2^2+3^2, 
a+b+c+d=1^2+2^2+3^2+4^2 =
sqrt(a)+sqrt(b)+sqrt(c)+sqrt(d)=1+2+3+4

Solução:
  Para a,b,c fixos, ponha x = d e analise f(x) = 
sqrt(a)+sqrt(b)+sqrt(c)+sqrt(x), 0=x=30-a-b-c. Essa eh uma funcao 
crescente, e portanto seu máximo ocorre quando x = 30-a-b-c, i.e, 
a+b+c+d=30.
  Agora troque c por x. Para a,b fixados, voce tem 0=x=14-a-b, d=30-x-a-b 
e olhando para
g(x) = sqrt(a)+sqrt(b)+sqrt(x)+sqrt(30-x-a-b),
2g'(x) = 1/sqrt(x) -1/sqrt(30-x-a-b)
   Observe que g eh crescente de x=0 ateh x=15-(a+b)/2. Como a+b0, 
14-(a+b)  15-(a+b)/2 e portanto o máximo dentro da restrição ocorre quando 
x=14-(a+b), i.e, a+b+c=14 e portanto d = 16.
   Agora voce tem um novo problema.. Basta mostrar que
   a=1^2,  a+b=1^2+2^2, a+b+c=1^2+2^2+3^2 = 
sqrt(a)+sqrt(b)+sqrt(c)=1+2+3
   Pronto, é só repetir o raciocínio para concluir que c=9, b=4 e a=1 dão o 
valor máximo da soma pedida.

Obs: Essa demonstração não pode ser adaptada fielmente para uma versão desse 
problema com 5 letras. Ficam então duas perguntas: Qual o maior valor de n 
tal que a_1+...+a_k =1^2+...+k^2 para k=1,2,..,n sempre implica 
sqrt(a1)+...+sqrt(an)=1+2+...+n?


- Original Message - 
From: Luiz Felippe medeiros de almeida [EMAIL PROTECTED]
To: obm-l@mat.puc-rio.br
Sent: Thursday, April 21, 2005 10:46 PM
Subject: Re: [obm-l] Duvida


Olá Fernado , acho q consegui fazer o problema que vc pediu. Lá vai:
a=1
a+b=5  == b=5-a == b=4 == sqrt(b)=2
a+b+c=14 == c= a+b == c= 14-4-1 ==sqrt(c)=3
a+b+c+d=30 == d=30-a-b-c== d=30-1-4-9 = sqrt(d)=4
Logo somando todas as equações temos :
  sqrt(a) + sqrt(b) + sqrt(c) + sqrt(d) =10
 Abraço
Luiz Felippe Medeiros
On 4/21/05, Fernando [EMAIL PROTECTED] wrote:
a = 1
a+b = 5
a+b+c = 14
a+b+c+d = 30
Prove: sqrt(a)+sqrt(b)+sqrt(c)+sqrt(d) = 10
Desde ja agradeço
[]'s
=
Instruções para entrar na lista, sair da lista e usar a lista em
http://www.mat.puc-rio.br/~nicolau/olimp/obm-l.html
=


=
Instruções para entrar na lista, sair da lista e usar a lista em
http://www.mat.puc-rio.br/~nicolau/olimp/obm-l.html
=


[obm-l] Livros para ITA/IME

2005-04-15 Por tôpico marcio aparecido
aeee galera tô estudando para as provas do ITA/IME, e venho aqui pedir
para que me indiquem   alguns livros de qiímica física e matemática.
Livros tanto para pegar uma base bem sólida como tmb para um belo
aprofundamento

=
Instruções para entrar na lista, sair da lista e usar a lista em
http://www.mat.puc-rio.br/~nicolau/olimp/obm-l.html
=


Re: [obm-l] soma de termos

2005-04-06 Por tôpico Marcio Cohen



Oi Cláudio.. Realmente é muito mais legal uma demonstração combinatória: 
Considere o conjunto dos números 0,1,2,3,...,n. Você quer escolher 
umsequencia a1  a2  ...  a(2m+1) de 2m+1 elementos, o que pode 
ser feito de "lado direito modos".Por outro 
lado, para cada k=0...n, voce pode escolher o elemento k como sendo o termo do 
meio dessa sequencia, e então precisa escolher binomial(k,m) termos menores e 
binomial(n-k,m) termos maiores que k. Somando em 
k, vemos que a resposta é o lado esquerdo e está provado.
 
 Mas não é tão feio fazer 
algebricamente..Vamos generalizar e provar que Soma(k=0..n) 
Binomial(k,a)*Binomial(n-k,b) = Binomial (n+1,a+b+1)

 Por inducao em n. Para n=0 eh facil. 
Supondo valido para n fixo e a,b quaisquer, temos:
Soma(k=0..n+1) Binomial(k,a)*binomial (n+1-k,b) = 
Soma(k=0..n) Binomial(k,a)*[Binomial(n-k,b)+Binomial(n-k,b-1)] + 
Binom(n+1,a)*Binom(0,b)
Usando a hipotese indutiva, isso da: 
Binomial(n+1,a+b+1) + Binomial(n+1, a+b) = Binomial (n+2, a+b+1)
 Em particular, fazendo a=b=m 
voce tem a solucao do problema pedido ;) (tá, confesso que tentei fazer a 
indução direto antes e não consegui :) E demorei bem menos pra dar a solução 
combinatória do que por indução.. mas não resisti ao "quero ver alguém ..." :) 

 Abraços,
 Marcio


  - Original Message - 
  From: 
  claudio.buffara 
  To: obm-l 
  Sent: Wednesday, April 06, 2005 3:58 
  PM
  Subject: Re: [obm-l] soma de termos
  
  Por exemplo, é possível dar uma demonstração combinatória da identidade 
  abaixo, que foi uma questão da famosa e difícil prova do IME de 1980/81.
  
  SOMA(k=0...n) Binom(k,m)*Binom(n-k,m) = Binom(n+1,2m+1).
  
  Agora, quero ver alguémprovar isso algebricamente...
  


[obm-l] Princípio das gavetas

2005-03-30 Por tôpico Marcio M Rocha
Olá, pessoal!
Antes de mais nada, obrigado ao Cláudio e ao Qwert pela solução do problema.
Como estou com um tempinho livre, vou escrever uns pensamentos muito 
rápido. Vejam se tem algum fundamento.

Em 39 números consecutivos, formo 13 conjuntos disjuntos, cada qual com 
3 números consecutivos. Obviamente, um deles é múltiplo de 3, o que 
implica que a soma dos algarismos de um elemento de cada um dos 13 
conjuntos é igual a 3k.

Tomando esse elemento de cada um dos 13 conjuntos, tenho 13 múltiplos 
consecutivos de 3, ou seja, 13 números cuja soma dos algarismos é um 
múltiplo de 3. Como são 13 números e todos são consecutivos, tô pensando 
se existe um meio de garantir que uma dessas somas também é múltiplo de 11.

O que vocês acham?
[]s,
Márcio.

=
Instruções para entrar na lista, sair da lista e usar a lista em
http://www.mat.puc-rio.br/~nicolau/olimp/obm-l.html
=


Re: [obm-l] Princípio das gavetas

2005-03-30 Por tôpico Marcio M Rocha
Marcio M Rocha escreveu:
Em 39 números consecutivos, formo 13 conjuntos disjuntos, cada qual 
com 3 números consecutivos. Obviamente, um deles é múltiplo de 3, o 
que implica que a soma dos algarismos de um elemento de cada um dos 13 
conjuntos é igual a 3k.

Tomando esse elemento de cada um dos 13 conjuntos, tenho 13 múltiplos 
consecutivos de 3, ou seja, 13 números cuja soma dos algarismos é um 
múltiplo de 3. Como são 13 números e todos são consecutivos, tô 
pensando se existe um meio de garantir que uma dessas somas também é 
múltiplo de 11.

O que vocês acham?
[]s,
Márcio.

=
Instruções para entrar na lista, sair da lista e usar a lista em
http://www.mat.puc-rio.br/~nicolau/olimp/obm-l.html
=
Eu respondo a mim mesmo: o fato dos números serem múltiplos consecutivos 
de 3 não significa que as somas dos algarismos sejam números consecutivos.

Desculpem-me.
[]s,
Márcio.
=
Instruções para entrar na lista, sair da lista e usar a lista em
http://www.mat.puc-rio.br/~nicolau/olimp/obm-l.html
=


Re: [obm-l] Problema 1 do primeiro nível da X Olimpíada de Maio

2005-03-30 Por tôpico Marcio M Rocha
Daniel S. Braz escreveu:
Pessoal,
Uma dúvida no problema 1 do primeiro nível da X Olimpíada de Maio
(Eureka! número 20)
Xavier multiplica quatro dígitos, não necessariamente distintos, e obtém um
número terminado em 7. Determine quanto pode valer a soma dos quatros dígitos
multiplicados por Xavier. Dê todas as possibilidades.
então devemos ter algo do tipo: a.b.c.d = xxx7
A dúvida é: Qualquer número terminado em 9 multiplicado por 3 dará um
número terminado em 7
1.1.1.7 = 7 - 10
1.1.3.9 = 27 - 14
1.1.3.19 = 57 - 24
1.1.3.29 = 87 - 34
1.1.3.39 = 117 - 44
Então...como calcular todas as possibilidades...não entendi...
 

Oi, Daniel
Veja que o enunciado diz que ele multiplica quatro dígitos, ou seja, 
algarismos. Assim, as 3 últimas possibilidades que você mostrou estão 
descartadas.

[]s,
Márcio.
=
Instruções para entrar na lista, sair da lista e usar a lista em
http://www.mat.puc-rio.br/~nicolau/olimp/obm-l.html
=


[obm-l] Principio das Gavetas

2005-03-29 Por tôpico Marcio M Rocha
Bom dia, pessoal!
Gostaria de conferir uma solução do seguinte problema: Mostre que 
existe um múltiplo de 1997 que possui todos os dígitos iguais a 1

Usando o princípio das gavetas é possível mostrar que todo número 
natural possui um múltiplo que se escreve usando apenas os dígitos 0 e 
1, de modo que haja uma seqüência de /p/ 1's seguida de /q/ 0's.

Seja N = 111...1000...0 um múltiplo de 1997. Como N = (111...1) * 
(10^/q/) e 1997 não divide 10^/q, /conclui-se que 1997 divide 111...1.

Tá tudo Ok?
Aproveitando a oportunidade, gostaria de uma sugestão no problema 
seguinte: Prove que em qualquer seqüência de 39 números naturais 
consecutivos existe ao menos um número cuja soma dos algarismos é 
divisível por 11.

[]s,
Márcio.
=
Instruções para entrar na lista, sair da lista e usar a lista em
http://www.mat.puc-rio.br/~nicolau/olimp/obm-l.html
=


Re: [obm-l] Principio das Gavetas

2005-03-29 Por tôpico Marcio M Rocha
claudio.buffara escreveu:
 
*De:* 	[EMAIL PROTECTED]

*Para:* obm-l@mat.puc-rio.br
*Cópia:*
*Data:* Tue, 29 Mar 2005 08:44:28 -0300
*Assunto:*  [obm-l] Principio das Gavetas
 Bom dia, pessoal!

 Gostaria de conferir uma solução do seguinte problema: Mostre que
 existe um múltiplo de 1997 que possui todos os dígitos iguais a 1

 Usando o princípio das gavetas é possível mostrar que todo número
 natural possui um múltiplo que se escreve usando apenas os dígitos 0 e
 1, de modo que haja uma seqüência de /p/ 1's seguida de /q/ 0's.

 Seja N = 111...1000...0 um múltiplo de 1997. Como N = (111...1) *
 (10^/q/) e 1997 não divide 10^/q, /conclui-se que 1997 divide 111...1.

 Tá tudo Ok?

Pra mim, está.
 
Uma demonstração alternativa usa o teorema de Euler e leva em conta 
que mdc(1997,10) = mdc(1997,9) = 1.
Nesse caso, pondo k = Phi(1997), teremos 10^k == 1 (mod 1997) ==
1997 | 10^k - 1 = 999999  (k algarismos 9) = 9*111...111.
Como 1997 é primo com 9, concluímos que 1997 | 111...111.
 
 
 Aproveitando a oportunidade, gostaria de uma sugestão no problema
 seguinte: Prove que em qualquer seqüência de 39 números naturais
 consecutivos existe ao menos um número cuja soma dos algarismos é
 divisível por 11.

Esse parece interessante. Acho que vale a pena fazer umas simulações 
no Excel pra ver se você acha alguma periodicidade ou lei de formação. 
Se eu achar alguma coisa te falo.
 
[]s,
Claudio.
 
Cláudio,
Obrigado pela solução alternativa e pela dica.
[]s,
Márcio.
=
Instruções para entrar na lista, sair da lista e usar a lista em
http://www.mat.puc-rio.br/~nicolau/olimp/obm-l.html
=


[obm-l] Problemas russos

2005-03-28 Por tôpico Marcio M Rocha
Bom dia a todos!
Encontrei 100 problemas russos traduzidos pelo Paulo Santa Rita e estou 
tentando resolvê-los. Gostaria de uma idéia para o seguinte:

É dado um retangulo ABCD com o comprimento da diagonal AC valendo L. 
Quatro círculos com centros em A, B, C e D  e raios respectivamente 
iguais a a, b, c e d, sao tais que : L  a + c ,  a + c = b + d. 
Prove que se pode inscrever um circulo no quadrilatero formado pelas 
interseccões entre duas tangentes comuns externas ao circulos A e C e 
duas tangentes comuns externas aos circulos B e D.

Um grande abraço.
[]s,
Márcio.
=
Instruções para entrar na lista, sair da lista e usar a lista em
http://www.mat.puc-rio.br/~nicolau/olimp/obm-l.html
=


[obm-l] questão física

2005-03-27 Por tôpico marcio aparecido
sejam dois trens de comprimentos m, correm em linha paralelas com
velocidades v1 e v2, (v2v1), no mesmo sentido, quanto tempo demora
para o trem mais rápido ultrapassar o mais lento?

=
Instruções para entrar na lista, sair da lista e usar a lista em
http://www.mat.puc-rio.br/~nicolau/olimp/obm-l.html
=


Re: [obm-l] ITA

2005-03-26 Por tôpico Marcio M Rocha
fgb1 escreveu:
Um aluno me pediu p/ fazer essa questão e disse que era do ITA. Não 
encntrei solução. Queria saber se alguem conhece e pode confirmar se o 
enunciado está correto.
 
3^2x + 5^2x - 15^x = 0
 
A idéia é dividir tudo por 15^x e, por meio de artifício, cair numa eq 
do 2o grau. Só que, fazendo tudo isso, você vai cair numa eq sem solução 
real. Deve haver algo de errado no enunciado.

[]s,
Márcio.
=
Instruções para entrar na lista, sair da lista e usar a lista em
http://www.mat.puc-rio.br/~nicolau/olimp/obm-l.html
=


[Fwd: Re: En:[obm-l] Fatorial]

2005-03-24 Por tôpico Marcio M Rocha
Desculpem a todos, mais uma vez, as duas besteiras que escrevi: a 
desigualdade absurda e minimilidade. No afã de resolver o problema, 
fiquei cego a algo tão claro.

Bem, então não dá para escrever uma seqüência de desigualdades partindo 
de b elevado a (b - 2) e chegando a (b - 1) elevado a (b - 1)?

[]s,
Márcio.
---BeginMessage---
claudio.buffara escreveu:

 Além disso, Cláudio, também posso fazer:

 b elevado a (b - 2) = (b - 1) elevado a (b - 2)
 
Epa! Tem certeza?
 
 
CARAMBA! QUE BESTEIRA! Deleta esse negócio daí!
---End Message---


  1   2   3   4   >